You are on page 1of 84

MISSION SHOURYA PSI TEST

QæAiÉÄÃnªï L.J.J¸ïSERIES
CPÁqÉ«Ä -2024
¨ÉAUÀ¼ÀÆgÀÄ
Student Name («zÁåyðAiÀÄ ºÉ¸ÀgÀÄ) ¥Àæ±Éß ¥ÀwæPÉ ±ÉæÃtÂ
Mobile No (ªÉƨÉÊ¯ï £ÀA§gï)

D
E-mail (E-ªÉÄïï)
Test batch No (¨ÁåZï ¸ÀASÉå)
UÀjµÀ× ¸ÀªÀÄAiÀÄ: 1 UÀAmÉ 30 ¤«ÄµÀUÀ¼ÀÄ mɸïÖ ¸ÀASÉå: 11 «µÀAiÀÄ ¸ÀAPÉÃvÀ: ¥ÀwæPÉ - 2 UÀjµÀ× CAPÀUÀ¼ÀÄ: 150

MISSION SHOURYA
PSI TEST SERIES
TEST NO – 11 ENGLISH SYNOPSIS
FOR KEY ANSWERS JOIN
OUR TELEGRAM CHANNEL

@ https://t.me/CREATIVEIAS

1
MISSION SHOURYA PSI TEST SERIES -2024
1. “Operation Chakra-II” recently seen in the news, is related to which of the following?
a) To fight against transnational organised cyber- enabled financial crimes in India.
b) To safeguard India’s ancient treasures from the Mauryan period.
c) To fight against naxal movement in India’s red corridor.
d) To facilitate the return of Indian nationals from war-torn Israel.
Answer: (a)
 The Operation Chakra-II was launched by the CBI to fight against transnational organised cyberenabled
financial crimes in India.
 For this, CBI has partnered with Microsoft and Amazon as well as with national and international
 agencies to combat and dismantle infrastructure of illegal call centres.

2. With reference to the APAAR, Automated Permanent Academic Account Registry, consider the following
statements :
1. APAAR is a part of the ‘One nation, One Student ID’ initiative of the Union government.
2. It is envisaged as an identity that gives students citizenship status.
Which of the statements given above are correct?
a) 1 only
b) 2 only
c) 1 and 2 only
d) None Of The Above
Answer: (a)
 Statement 1 and 2 are correct: APAAR (Automated Permanent Academic Account Registry) is a part
of the ‘One nation, One Student ID’ initiative of the Union government.
 It is envisaged as a new student identity card launched as part of the National Education Policy 2020.
 There is no provision regarding giving students citizenship status.
 To sign up for APAAR, students will have to provide basic information such as name, age, date of birth,
gender, and a photograph. This information will be verified using their Aadhar number.
 Students will need to sign a consent form, and they can choose to either accept or decline sharing their
Aadhar number and demographic information with the Ministry of Education for creating the APAAR
ID.
 For minors, parents will have to sign the consent form, allowing the Ministry to use the student’s
Aadhar number for authentication with UIDAI.
 Registration for creating APAAR ID is voluntary, not mandatory.

3. D K Mittal Committee is associated with which of the following?


a) Privatisation of Banks
b) Reforms to achieve the developed economy status by 2047.
c) Action plan for revenue generation of Indian Railways
d) Prisons reforms
Answer: (c)
 Indian Railways (IR) has been splurging on capital expenditures (capex), there have been concerns
about its operating ratio and revenue generation.
 Indian Railways is an extensive network covering over 67,000 route kilometers and serving around
8,000 stations.
 It is the world’s 4th largest railway network, followed by the US, China and Russia.
 It is often referred to as the “lifeline of India” due to its crucial role in transporting goods and
passengers across the country.
 D. K. Mittal Committee was formed in 2014 to provide recommendations on raising revenue for the
Indian Railways.
 Fare Rationalization: Periodic review and adjustment of passenger fares to better reflect the operational
costs.

2
MISSION SHOURYA PSI TEST SERIES -2024
 Implementing dynamic pricing to vary fares based on demand and time of booking, particularly for
premium trains.
 Freight Tariff Reform: Simplifying the freight tariff structure to make it more competitive and
transparent.
 Offering discounts and incentives for high-volume cargo and long-term freight contracts.
 Monetization of Assets: Exploring opportunities to monetize railway land and properties through
commercial development, leasing, or joint ventures.
 Developing railway stations as commercial hubs with retail spaces, restaurants, and entertainment
options, on the lines of National Monetization Pipeline.

4. Consider the following statements with reference to the Nobel Prize in Economic Sciences:
1. In 2023, it was awarded to Narges Mohammadi for providing the first comprehensive account of women’s
earnings.
2. It was given first time to Ragnar Frisch and Jan Tinbergen in 1969.
Which of the statements given above are correct?
a) 1 only
b) 2 only
c) 1 and 2 only
d) None Of The Above
Answer: (b)
 Statement 1 is not correct : The Nobel Prize in Economics for 2023 was awarded to Harvard
University professor Claudia Goldin for research that has advanced the understanding of the gender gap
in the labour market.
 Claudia Goldin became the third woman to receive the Nobel Prize in economics
 She provided the first comprehensive account of women’s earnings and labour market participation
through the centuries.
 Statements 2 and 3 are correct : In 1968, Sveriges Riksbank (Sweden’s central bank) established the
Prize in Economic Sciences in Memory of Alfred Nobel, founder of the Nobel Prize.
 The first prize in economic sciences was awarded to Ragnar Frisch and Jan Tinbergen in 1969.
 It is officially known as The Sveriges Riksbank Prize in Economic Sciences
 The prize in economic sciences is awarded by the Royal Swedish Academy of Sciences, Stockholm,
Sweden, according to the same principles as for the Nobel Prizes that have been awarded since 1901.
 It has been awarded 55 times to 93 laureates between 1969 and 2023.

5. With reference to Asian Games, consider the following statements:


1. The 19th edition of the Asian games was recently held in Seoul, South Korea.
2. India holds the record for holding most of the editions of the game since its inception in 1951.
Select the correct answer using the code given below:
a) 1 only
b) 2 only
c) 1 and 2 only
d) None Of The Above
Answer: (d)
 Asian Games: 2023 Edition (Originally scheduled for 2022) was held in Hangzhou, People’s Republic
of China, from 23 September to 8 October, 2023.
 Thailand boasts an impressive record of hosting Asian games events for four times- 1966, 1970, 1978,
and 1998, while India hosted only two editions: 1951 and 1982.
 The Asian Games, also known as Asiad, were regulated by the Asian Games Federation from 1951 to
1978 and since 1982, Olympic Council of Asia regulates the Asian Games. Hence, statement 3 is

3
MISSION SHOURYA PSI TEST SERIES -2024
correct.

6. Consider the following statements


Statement I : State emergency is called as “President Rule”
Statement II : Centre takes over the government of the state under Article 356 in case of failure of
constitutional machinery in state
Which of the statements given above are correct?
a) Both Statement I and Statement II correct and statement II is correct explanation for Statement I
b) Both Statement I and Statement II correct and statement II is not correct explanation for Statement I
c) Statement I is correct but Statement II is incorrect
d) Statement I is incorrect but Statement II is correct
Answer: A
 It is this duty in the performance of which the Centre takes over the government of a state under Article
356 in case of failure of constitutional machinery in state. This is popularly known as ‘President’s
Rule’. It is also known as ‘State Emergency’ or ‘Constitutional Emergency’. Hence Both Statement I
and Statement II correct and statement II is correct explanation for Statement I
Additional Information
The President’s Rule can be proclaimed under Article 356 on two grounds
 One mentioned in Article 356 itself and another in Article 365:
1. Article 356 empowers the President to issue a proclamation, if he is satisfied that a situation has
arisen in which the government of a state cannot be carried on in accordance with the provisions of
the Constitution. Notably, the president can act either on a report of the governor of the state or
otherwise too (ie, even without the governor’s report).
2. Article 365 says that whenever a state fails to comply with or to give effect to any direction from
the Centre, it will be lawful for the president to hold that a situation has arisen in which the
government of the state cannot be carried on in accordance with the provisions of the Constitution.
 Parliamentary Approval and Duration
o President’s Rule must be approved by both the Houses of Parliament within two months from the
date of its issue.
o If the proclamation of President’s Rule is issued at a time when the Lok
o Sabha has been dissolved or the dissolution occurs during the period of two months without
approving the proclamation, then it survives until 30 days from the first sitting of the Lok Sabha,
meanwhile Rajya Sabha approves it in that time.
o If approved by both the Houses of Parliament, the President’s Rule continues for six months
o It can be extended for a maximum period of three years 7 with the approval of the Parliament,
every six months.
o Every resolution approving the proclamation of President’s Rule or its continuation can be passed
by either House of Parliament only by a simple majority
New Provision
44th Amendment Act of 1978 introduced a new provision that beyond one year, the President’s Rule can be
extended by six months at a time only when the following two conditions are fulfilled:
o Proclamation of National Emergency should be in operation in the whole of India, or in the whole
or any part of the state
o Election Commission must certify that the general elections to the legislative assembly of the
concerned state cannot be held on account of difficulties.
Consequences of President’s Rule
4
MISSION SHOURYA PSI TEST SERIES -2024
The President acquires the following extraordinary powers during President’s Rule is imposed in a state:
1. He can take up the functions of the state government and powers vested in the governor or any other
executive authority in the state.
2. He can declare that the powers of the state legislature are to be exercised by the Parliament.
3. He can take all other necessary steps including the suspension of the constitutional provisions relating to
any body or authority in the state.
 The state governor, on behalf of the President, carries on the state administration with the help of the
chief secretary of the state or the advisors appointed by the President.
 When President’s Rule is imposed in a state, he dismisses the state council of ministers headed by the
chief minister.
When the state legislature is suspended or dissolved:
1. Parliament can delegate the power to make laws for the state to the President or to any other authority
specified by him in this regard,
2. Parliament or in case of delegation, the President or any other specified authority can make laws
conferring powers and imposing duties on the Centre or its officers and authorities,
3. President can authorize, when the Lok Sabha is not in session, expenditure from the state consolidated
fund pending its sanction by the Parliament, and
4. President can promulgate, when the Parliament is not in session, ordinances for the governance of the
state.
Note : Constitutional position, status, powers and functions of the concerned state high court remain same
even during the President’s Rule.

7. Consider the following statements


1. The Supreme Court is empowered to issue writs including habeas corpus, mandamus, prohibition,
quo-warranto, and certiorari which is appropriate for enforcement of Fundamental Rights
2. Writ jurisdiction of the Supreme Court is exclusive.
Which of the above mentioned statement/ statements is /are correct
a) Only 1
b) Only 2
c) Both of them
d) None of them
Answer: A
 The Supreme Court is empowered to issue writs including habeas corpus, mandamus, prohibition, quo
warranto and certiorari for the enforcement of the fundamental rights of an aggrieved citizen. Hence
statement 1 is correct
 Writ jurisdiction of the Supreme Court is not exclusive. The high courts are also empowered to issue
writs for the enforcement of the Fundamental Rights. Hence Statement 2 is Incorrect
Additional Information
 SC has original jurisdiction in the sense that an aggrieved citizen can directly go to the Supreme Court,
not necessarily by way of appeal.
 When the Fundamental Rights of a citizen are violated, the aggrieved party has the option of moving
either the high court or the Supreme Court directly.
 The original jurisdiction of the Supreme Court with regard to federal disputes is different from its
original jurisdiction with regard to disputes relating to fundamental rights. In the first case, it is
exclusive and in the second case, it is concurrent with high courts jurisdiction.
 Parties involved in the federal cases are units of the federation (Centre and states) while the dispute in
the second case is between a citizen and the Government (Central or state).
Writ Jurisdiction
 There is a difference between the writ jurisdiction of the Supreme Court and that of the high court.
 The Supreme Court can issue writs only for the enforcement of the Fundamental Rights and not for
5
MISSION SHOURYA PSI TEST SERIES -2024
other purposes.
 The high court, on the other hand, can issue writs not only for the enforcement of the fundamental rights
but also for other purposes.
 Writ jurisdiction of the high court is wider than that of the Supreme Court. But, the
 Parliament can confer on the Supreme Court, the power to issue writs for other purposes also.

8. The Fundamental Rights, enshrined in Part III of the Constitution of India, are inherent and
cannot be extinguished by any constitutional and statutory provision. This was made clear in the
case of
a) K. Gopalan v/s State of Madras
b) I C Golak Nath v/s State of Punjab
c) State of West Bengal v/s Committee for Protection of Democratic Rights, West Bengal
d) Shankari Prasad Singh Deo v/s Union of India
Answer: B
 In the 1967 Golak Nath v. State of Punjab case, the Supreme Court reversed its previous rulings in the
Shankari Prasad and Sajjan Singh cases, asserting that Fundamental Rights were not amenable to
amendment through the constitutional procedure outlined in Article 368. Hence Option B is correct
 In response, Parliament enacted the 24th Constitutional Amendment Act in 1971 to nullify the Supreme
Court's decision.
 24th amendment explicitly stated that Parliament had the authority to amend any portion of the
Constitution, a stance upheld by the Supreme Court in the Kesavananda Bharati case in 1973.
Important Cases
A.K. Gopalan v. State of Madras  The Court ruled that protection under Article 21 is
available only against arbitrary executive action and
not from arbitrary legislative action.
State of West Bengal V. Committee for  The Supreme Court in the present case gave a wide
Protection of Democratic Rights, West interpretation to the power of judicial review conferred
Bengal upon it by Article 32 and upon High Courts under Article
226 of the Constitution.
 The separation of power between the Parliament and the
State Legislature was also discussed at length.
Shankari Prasad Singh Deo v. Union of  Judgement laid that Parliament to amend the Constitution
India under Article 368 also includes the power to amend
Fundamental Rights.
 Word ‘law’ in Article 13 includes only ordinary laws and
not the constitutional amendment acts (constituent laws)
 Parliament can abridge or take away any of the
Fundamental Rights by enacting a constitutional
amendment act and such a law will not be void under
Article 13.
S.R. Bommai and Ors. vs. Union of  Court discussed at length provisions of Article 356 of the
India Constitution of India and related issues.
 Power of the President to dismiss a State government is
not absolute.
 President should exercise the power only after his
proclamation (imposing his/her rule) is approved by both
Houses of Parliament.
 Presidential Proclamation under Article 356 is subject to
judicial review.
Berubari Union Case, 1960  Supreme Court declared that the Preamble is not a part of

6
MISSION SHOURYA PSI TEST SERIES -2024
the Constitution
 Established that the Indian Parliament has the power to
amend the Constitution
L Chandra Kumar Case (1997)  Articles 32 (Right to Constitutional Remedies) and 226
(Judicial Review Power) of the Constitution are
fundamental elements of the Constitution
Golaknath v the State of Punjab (1967)  Supreme Court declared that Fundamental Rights could
not be amended by the Parliament even for
implementation of Directive Principles.
Champakam Dorairajan v/s State of  Supreme Court ruled that in case of any conflict between
Madras the Fundamental Rights and the Directive Principles, the
former would prevail.
 Directive Principles have to conform to and run as
subsidiary to the Fundamental Rights.
 Fundamental Rights could be amended by the Parliament
by enacting constitutional amendment acts.
Minerva Mills Case:  Parliament can amend any part of the Constitution but it
cannot change the “Basic Structure” of the Constitution.
Kesavananda Bharati case  Supreme Court overruled its Golak Nath (1967) verdict
and declared that Parliament can amend any part of the
Constitution but it cannot alter its “Basic Structure”.
Shah Bano Case  Supreme Court upheld the right of a Muslim woman to
claim maintenance from her husband even after the Iddat
period
Indira Sawhney Case  Upheld the 27% quota for backward classes and struck
down the government notification reserving 10%
government jobs for economically backward classes
among the higher castes.
 Principle that the combined reservation beneficiaries
should not exceed 50% of India’s population.
 Concept of ‘creamy layer’ also gained importance
Waman Rao Case  Basic Structure theory was reaffirmed by the SC
MC Mehta and Union Of India  SC ruled that its authority under Article 32 extends to
corrective actions as well as preventive ones where rights
are violated
I.R Coelho and State of Tamil Nadu  Laws that are listed in the Indian Constitution’s 9th
Schedule are nevertheless subject to scrutiny and
confrontation in court.
 There is a list of acts and laws that cannot be contested in
court in the 9th Schedule of the Indian Constitution
Lily Thomas and Union Of India  SC held that any MLA, MLC, or MP who was convicted
of a crime and sentenced to at least two years in jail
would immediately lose their right to serve in the House
Puttaswamy case  Upheld that each individual has a basic right to privacy
under the Indian Constitution which is mentioned in
Article 21.

7
MISSION SHOURYA PSI TEST SERIES -2024
Vishaka case  Judgment passed by the supreme court is a significant
investigation concerning a woman's sexual assault at her
place of employment

9. Match List I (Item in the Indian Constitution) with List II (Country from which it was derived) and
select the correct answer using the codes given below the list.
(Item in the India (Country)
Consti tution)
A. Directive Principles 1. Australia
of State Policy
B. Fundamental Rights 2. Canada
C. Concurrent List in 3. Ireland
Union-State
Relations
D. India as a Union of 4. United
States with greater kingdom
powers to the Union
5. USA
Codes
a) A-5, B-4,C-1,D-2
b) A-3, B-5,C-2, D-1
c) A-5, B -4,C-2,D-1
d) A-3, B-5,C-1,D-2
Answer: D

Sources Features Borrowed


Government of India  Federal Scheme, Office of governor, Judiciary, Public Service
Act of 1935 Commissions, Emergency provisions and administrative details.
British Constitution  Parliamentary government, Rule of Law, legislative procedure,
single citizenship, cabinet system, prerogative writs, parliamentary
privileges and bicameralism.
US Constitution  Fundamental rights, independence of judiciary, judicial review,
impeachment of the president, removal of Supreme Court and high
court judges and post of vice-president.
Irish Constitution  Directive Principles of State Policy, nomination of members to
Rajya Sabha and method of election of president.
Canadian Constitution  Federation with a strong Centre, vesting of residuary powers in the
Centre, appointment of state governors by the Centre, and advisory
jurisdiction of the Supreme Court.
Australian Constitution  Concurrent List, freedom of trade, commerce and inter-course, and
joint sitting of the two Houses of Parliament.
Weimar Constitution of  Suspension of Fundamental Rights during Emergency.
Germany
Soviet Constitution  Fundamental duties and the ideal of justice (social, economic and
(USSR, now Russia) political) in the Preamble.
French Constitution  Republic and the ideals of liberty, equality and fraternity in the
Preamble.
South African  Procedure for amendment of the Constitution and election of
Constitution members of Rajya Sabha.
Japanese Constitution  Procedure established by Law.

8
MISSION SHOURYA PSI TEST SERIES -2024
10. When was Panchayats (Extension to the schedule Areas) Act was passed by Indian Parliament?
a) 1998
b) 1996
c) 1995
d) 1993
Answer: B
 “Provisions of the Panchayats (Extension to the Scheduled Areas) Act”, 1996, popularly known as the
PESA Act or the Extension Act. Hence Option B is correct
Additional Information
 The provisions of Part IX of the constitution relating to the Panchayats are not applicable to the Fifth
Schedule areas.
 Parliament may extend these provisions to such areas, subject to such exceptions and modifications as it
may specify.
 Under Panchayat act provision, the Parliament has enacted PESA Act or the Extension Act.
States covered under act
 At present ten states have Fifth Schedule Areas.
 These states are: Andhra Pradesh, Telangana, Chhatisgarh, Gujarat, Himachal Pradesh, Jharkhand,
Madhya Pradesh, Maharashtra, Odisha and Rajasthan.
 All the ten states have enacted requisite compliance legislations by amending the respective Panchayati
Raj Acts.
Objectives of the Act
 To extend the provisions of Part IX of the Constitution relating to the panchayats to the scheduled areas
with certain modifications
 To provide self-rule for the bulk of the tribal population
 To have village governance with participatory democracy and to make the gram sabha a nucleus of all
activities
 To evolve a suitable administrative framework consistent with traditional practices
 To safeguard and to preserve the traditions and customs of tribal communities
 To empower panchayats at the appropriate levels with specific powers conducive to tribal requirements
 To prevent panchayats at the higher level from assuming the powers and authority of panchayats at the
lower level of the gram sabha
Features of the Act
 A state legislation on the Panchayats in the Scheduled Areas shall be in consonance with the customary
law, social and religious practices and traditional management practices of community resources.
 A village shall ordinarily consist of a habitation or a group of habitations or a hamlet or a group of
hamlets comprising a community and managing its affairs in accordance with traditions and customs.
 Every village shall have a Gram Sabha consisting of persons whose names are included in the electoral
rolls for the Panchayat at the village level.
 Every Gram Sabha shall be competent to safeguard and preserve the traditions and customs of the
people, their cultural identity, community resources and the customary mode of dispute resolution.
 Every Gram Sabha shall–
 (i) approve of the plans, programmes and projects for social and economic development before they are
taken up for implementation by the Panchayat at the village level;
 (ii) be responsible for the identification of beneficiaries under the poverty alleviation and other
programmes.
9
MISSION SHOURYA PSI TEST SERIES -2024
 Every Panchayat at the village level shall be required to obtain from the Gram Sabha a certification of
utilisation of funds for the above plans, programmes and projects.
 The reservation of seats in the Scheduled Areas in every Panchayat shall be in proportion to the
population of the communities for whom reservation is sought to be given under Part IX of the
Constitution.
 The reservation for the Scheduled Tribes shall not be less than one-half of the total number of seats.
Further, all seats of Chairpersons of Panchayats at all levels shall be reserved for the Scheduled Tribes.
 The state government may nominate such Scheduled Tribes which have no representation in the
Panchayat at the intermediate level or the Panchayat at the district level. But such nomination shall not
exceed one-tenth of the total members to be elected in that Panchayat.
 The Gram Sabha or the Panchayats at the appropriate level shall be consulted before making the
acquisition of land in the Scheduled Areas for development projects and before resettling or
rehabilitating persons affected by such projects in the Scheduled Areas.
 Planning and management of minor water bodies in the Scheduled Areas shall be entrusted to
Panchayats at the appropriate level.
 The recommendations of the Gram Sabha or the Panchayats at the appropriate level shall be mandatory
for grant of prospecting licence or mining lease for minor minerals in the Scheduled Areas.
 The prior recommendation of the Gram Sabha or the Panchayats at the appropriate level shall be
mandatory for grant of concession for the exploitation of minor minerals by auction.
 While endowing Panchayats in the Scheduled Areas with such powers and authority as may be
necessary to enable them to function as institutions of self-government
 State Legislature shall ensure that the Panchayats at the appropriate level and the Gram Sabha are
endowed specifically with–
i. The Power To Enforce Prohibition Or To Regulate Or Restrict The Sale And Consumption Of
Any Intoxicant
ii. The Ownership Of Minor Forest Produce
iii. The Power To Prevent Alienation Of Land In The Scheduled Areas And To Take Appropriate
Action To Restore Any Unlawfully Alienated Land Of A Scheduled Tribe
iv. The Power To Manage Village Markets
v. The Power To Exercise Control Over Money Lending To The Scheduled Tribes
vi. The Power To Exercise Control Over Institutions And Functionaries In All Social Sectors
vii. The Power To Control Local Plans And Resources For Such Plans Including Tribal Sub-Plans
 The State Legislations shall contain safeguards to ensure that Panchayats at the higher level do not
assume the powers and authority of any Panchayat at the lower level or of the Gram Sabha.
 The State Legislature shall endeavour to follow the pattern of the Sixth Schedule to the Constitution
while designing the administrative arrangements in the Panchayats at district levels in the Scheduled
Areas.

11. On which plateau, the tropic of cancer and the Indian standard time line intersect each other?
a) Bundelkhand
b) Baghelkhand
c) Malwa
d) Chotanagpur Plateau
Answer: B
 Tropic of Cancer (23.30′) passes through these 8 states in India: Gujarat, Rajasthan, Madhya Pradesh,
Chattisgarh, Jharkhand, West Bengal, Tripura, and Mizoram and also Vindhya Region.
 The standard meridian of India (82.30′) passes through five states in India which are Orissa,

10
MISSION SHOURYA PSI TEST SERIES -2024
Chattisgarh, Uttar Pradesh, Andhra Pradesh, and Madhya Pradesh.
 They both intersect at Koriya district in Chhattisgarh.. Hence B is correct
Additional Information

Peninsular Plateau
 Roughly triangular in shape with its base coinciding with the southern edge of the great plain of North
India. Apex of the triangular plateau is at Kanniyakumari.
 It covers a total area of about 16 lakh sq km (India as a whole is 32 lakh sq km).
 The average height of the plateau is 600-900 m above sea level (varies from region to region).
 Most of the peninsular rivers flow west to east indicating it’s general slope.
 Narmada-Tapti are the exceptions which flow from east to west in a rift (rift is caused by divergent
boundary (Go back to Interaction of plates).
 The Peninsular Plateau is a one of the oldest landforms of earth.
 It is a highly stable block composed mostly of the Archaean gneisses and schists {Rock System}.
 It has been a stable shield which has gone through little structural changes since its formation.
 Since few hundred million years, Peninsular block has been a land area and has never been submerged
beneath the sea except in a few places.
 Peninsular Plateau is an aggregation of several smaller plateaus, hill ranges interspersed with river
basins and valleys.
Marwar Plateau or Mewar Plateau
 It is the plateau of eastern Rajasthan. [Marwar plain is to the west of Aravalis whereas Marwar plateau
is to the east].
 The average elevation is 250-500 m above sea level and it slopes down eastwards.
 It is made up of sandstone, shales and limestones of the Vindhayan period.
 The Banas river, along with its tributaries [Berach river, Khari rivers] originate in the Aravali Range
and flow towards northwest into Chambal river. The erosional activity of these rives make the plateau
top appear like a rolling plain.
Central Highland
11
MISSION SHOURYA PSI TEST SERIES -2024
 Also called the Madhya Bharat Pathar or Madhya Bharat Plateau.
 It is to the east of the Marwar or Mewar Upland.
 Most of plateau comprises the basin of the Chambal river which flows in a rift valley.
 The Kali Sindh, flowing from Rana Prataph Sagar, The Banas flowing through Mewar plateau and The
Parwan and the Parbati flowing from Madhya Pradesh are its main tributaries.
 It is a rolling plateau with rounded hills composed of sandstone. Thick forests grow here.
 To the north are the ravines or badlands of the Chambal river [They are typical to Chambal river basin]{
Arid landforms}.
Bundelkhand Upland
 Yamuna river to the north, Madhya Bharat Pathar to the west, Vindhyan Scarplands to the east and
south-east and Malwa Plateau to the south.
 It is the old dissected (divided by a number of deep valleys) upland of the ‘Bundelkhand gneiss’
comprising of granite and gneiss.
 Spreads over five districts of Uttar Pradesh and four districts of Madhya Pradesh.
 Average elevation of 300-600 m above sea level, this area slopes down from the Vindhyan Scarp toward
the Yamuna River.
 The area is marked by a chain of hillocks (small hill) made of granite and sandstone.
 The erosional work of the rivers flowing here have converted it into an undulating (wave like surface)
area and rendered it unfit for cultivation.
 The region is characterized by senile (characteristic of or caused by old age) topography.
 Streams like Betwa, Dhasan and Ken flow through the plateau.
Malwa Plateau
 The Malwa Plateau roughly forms a triangle based on the Vindhyan Hills, bounded by the Aravali
Range in the west and Madhya Bharat Pathar to the north and Bundelkhand to the east.
 This plateau has two systems of drainage; one towards the Arabian sea (The Narmada, the Tapi and the
Mahi), and the other towards the Bay of Bengal (Chambal and Betwa, joining the Yamuna).
 In the north it is drained by the Chambal and many of its right bank tributaries like the Kali, the Sindh
and the Parbati. It also includes the upper courses of the Sindh, the Ken and the Betwa.
 It is composed of extensive lava flow and is covered with black soils.
 The general slope is towards the north [decreases from 600 m in the south to less than 500 m in the
north]
 This is a rolling plateau dissected by rivers. In the north, the plateau is marked by the Chambal ravines.
Baghelkhand
 North of the Maikal Range is the Baghelkhand.
 Made of limestones and sandstones on the west and granite in the east.
 It is bounded by the Son river on the north.
 The central part of the plateau acts as a water divide between the Son drainage system in the north and
the Mahanadi river system in the south.
 The region is uneven with general elevation varying from 150 m to 1,200 m.
 The Bhanrer and Kaimur are located close to the trough-axis.
 The general horizontality of the strata shows that this area has not undergone any major disturbance.
Chotanagpur Plateau
 Chotanagpur plateau represents the north-eastern projection of the Indian Peninsula.
 Mostly in Jharkhand, northern part of Chhatisgarh and Purulia district of West Bengal.
12
MISSION SHOURYA PSI TEST SERIES -2024
 The Son river flows in the north-west of the plateau and joins the Ganga.
 The average elevation of the plateau is 700 m above sea level.
 This plateau is composed mainly of Gondwana rocks.
 The plateau is drained by numerous rivers and streams in different directions and presents a radial
drainage pattern. {Drainage Pattern}
 Rivers like the Damodar, the Subarnrekaha, the North Koel, the South Koel and the Barkar have
developed extensive drainage basins.
 The Damodar river flows through the middle of this region in a rift valley from west to east. Here are
found the Gondwana coal fields which provide bulk of coal in India.
 North of the Damodar river is the Hazaribagh plateau with an average elevation of 600 m above mean
sea level. This plateau has isolated hills. It looks like a peneplain due to large scale erosion.
 The Ranchi Plateau to the south of the Damodar Valley rises to about 600 m above mean sea level.
Most of the surface is rolling where the city of Ranchi (661 m) is located.
 At places it is interruped by monadnocks (an isolated hill or ridge of erosion-resistant rock rising above
a peneplain. Ex: Ayers Rock in Australia) and conical hills.
 The Rajmahal Hills forming the north eastern edge of the Chotanagpur Plateau are mostly made of
basalt and are covered by lava flows {Basaltic Lava}.
 They run in north-south direction and rise to average elevation of 400 m (highest mount is 567 m).
These hills have been dissected into separate plateaus.
Meghalaya Plateau
 The peninsular plateau extends further east beyond the Rajmahal hills to from Meghalaya or the
Shillong plateau.
 Garo-Rajmahal Gap separates this plateau from the main block.
 This gap was formed by down-faulting (normal fault: a block of earth slides downwards). It was later
filled by sediments deposited by the Ganga and Brahmaputa.
 The plateau is formed by Archaean quartzites, shales and schists.
 The plateau slopes down to Brahmaputra valley in the north and the Surma and Meghna valleys in the
south.
 Its western boundary more or less coincides with the Bangladesh border.
 The western, central and the eastern parts of the plateau are known as the Garo Hills (900 m), the Khasi-
Jaintia Hills (1,500 m) and the Mikir Hills (700 m).
 Shillong (1,961 m) is the highest point of the plateau.
Deccan Plateau
 It covers an area of about five lakh sq km.
 It is triangular in shape and is bounded by the Satpura and the Vindhya in the north-west, the Mahadev
and the Maikal in the north, the Western Ghats in the west and the Eastern Ghats in the east.
 Its average elevation is 600 m.
 It rises to 1000 m in the south but dips to 500 m in the north.
 Its general slope is from west to east which is indicated by the flow of its major rivers.
 Rivers have further subdivided this plateau into a number of smaller plateaus.
Maharashtra Plateau
 The Maharashtra Plateau lies in Maharashtra.
 It forms the northern part of the Deccan Plateau.
 Much of the region is underlain by basaltic rocks of lava origin [Most of the Deccan Traps lies in this

13
MISSION SHOURYA PSI TEST SERIES -2024
region].
 The area looks like a rolling plain due to weathering.
 The horizontal lava sheets have led to the formation of typical Deccan Trap topography [step like].
 The broad and shallow valleys of the Godavari, the Bhima and the Krishna are flanked [bordered on the
opposite sides] by flat-topped steep sided hills and ridges.
 The entire area is covered by black cotton soil known as regur.
Karnataka Plateau
 The Karnataka Plateau is also known as the Mysore plateau.
 Lies to the south of the Maharashtra plateau.
 The area looks like a rolling plateau with an average elevation of 600-900 m.
 It is highly dissected by numerous rivers rising from the Western Ghats.
 The general trend of the hills is either parallel to the Western Ghats or across it.
 The highest peak (1913 m) is at Mulangiri in Baba Budan Hills in Chikmaglur district.
 The plateau is divided into two parts called Malnad and Maidan.
 The Malnad in Kannada means hill country. It is dissected into deep valleys covered with dense forests.
 The Maidan on the other hand is formed of rolling plain with low granite hills.
 The plateau tapers between the Western Ghats and the Eastern Ghats in the south and merges with the
Niligiri hills there.
Telangana plateau
 The Telangana plateau consists of Archaean gneisses.
 It’s average elevation is 500-600 m.
 The southern part is higher than its northern counterpart.
 The region is drained by three river systems, the Godavari, the Krishna and the Penneru.
 The entire plateau is divided into Ghats and the Peneplains (a vast featureless, undulating plain which
the last stage of deposition process).
Chhattisgarh Plain
 The Chhattisgarh plain is the only plain worth the name in the Peninsular plateau.
 It is a saucer shaped depression drained by the upper Mahanadi.
 The whole basin lies between the Maikala Range and the Odisha hills.
 The region was once ruled by Haithaivanshi Rajputs from whose thirty six forts (Chhattisgarh) it derives
its name.
 The basin is laid with nearly horizontal beds of limestone and shales.
 The general elevation of the plain ranges from 250 m in the east to 330 m in the west.

12. Consider the following statements:


Statement I: Tides are the rise and fall of sea levels caused by the combined effects of the gravitation
forces exerted by the Moon and the Sun and the rotation of the Earth.
Statement II : The Earth rotates from west to east.
Examine these two statements carefully and select the correct answer using the code given below
a) Both the statements are individually true, and Statement II is the correct explanation of Statement I
b) Both the statements are individually true, but Statement II is not the correct explanation of
Statement I
c) Statement I is true but Statement II is false
d) Statement I is false but Statement II is true
Answer: B
 Tides are the rhythmic rise and fall of ocean water that occur twice in a day. The strong gravitational

14
MISSION SHOURYA PSI TEST SERIES -2024
pull exerted by the sun and the moon on the earth's surface causes the tides. Hence Statement 1 is
correct
 The direction of earth rotation is from west to east. Hence statement 2 is individually correct but its not
the correct explanation to statement 1
Additional Information
Reasons for the formation of tides
 The gravitational force exerted on Earth by the Sun and the Moon
 Rotation of the Earth
 Centrifugal Force
Types of tides
Tides types as per the frequency
 Mixed Tide: When tides vary in height, they are known as Mixed Tides. These types of tides are
usually seen on the Pacific Ocean islands and along North America’s west coast.
 Diurnal Tide: A high tide and low tide are observed every day. The consecutive low or high tides are
around the same height.
 Semi-diurnal Tide: Semi-diurnal Tide pattern is commonly observed that features 2 high and 2 low
tides every day. The consecutive low or high tides are around the same height.

Tides types as per positions of the Earth, Moon, and Sun


 Spring Tides: When earth, moon, and sun fall on a straight line, high tides are observed. They occur
twice every month on the full and new moon days
 Neap Tides: When the positions of the moon and sun create a right angle, their gravitational forces
balance each other. Moon’s gravitational pull is twice as strong as compared to the sun’s pull. However,
it gets minimised by the sun’s force that counteracts it
 Usually, the gap between neap and spring tides is 7 days.
Tidal Range
 Tidal range Tidal ranges are high when the earth is nearest to the sun. This phenomenon known as
perihelion occurs every year somewhere near the 3rd of January. Unusual high and low tides are
experienced during this period
 Tidal ranges are low as compared to the average when the earth is farthest from the sun. This

15
MISSION SHOURYA PSI TEST SERIES -2024
phenomenon is known as aphelion and it occurs every year somewhere near the 4th of July

13. Which of the following districts shares border with the states Tamil nadu and Andhra Pradesh?
a) Chikkaballapura
b) Kolar
c) Tumkur
d) Ramnagar
Answer: B
 Kolar: shares border with Andra Pradesh and Tamil Nadu. Hence option B is correct
Belagavi Shares border with Maharashtra and Goa
Bidar Shares border with Telangana and Maharashtra
Gulbarga Shares border with Telangana and Maharashtra
Chamarajnagar Kerala and Tamil Nadu
Raichur Telangana and Andra Pradesh

14. Consider the following


1. Languli falls
2. Hogenikal falls
3. Magodu fals
4. Bhandaje falls
What is the correct sequence of these waterfalls in the descending order of their height
a) A,D,C,B
b) D,C,A,B
c) B,C,A,D
d) D,A,C,B
Answer : B

16
MISSION SHOURYA PSI TEST SERIES -2024

Kunchikal Falls Shimoga District Varahi river 455 metres


Barkana Falls Shimoga District River Sita 259 metres
Jog Falls Shimoga District Sharavathi 253 metres
Bhandaje falls Dakshina Kannada Netravati 216 metres
district
Magod Falls Uttara Kannada River Bedti 200 metres
Belkal Theertha Udupi District Hemavati river 180 metres
Hebbe Falls Chikmagalur District River Bhadra 168 metres
Kalhatti Falls Chikmagalur River Sharavath 123 metres
Chaya Bhagavathi falls Yadgir district River Doni 122 meters
Koosalli Falls Udupi District 120 metres
Keppa Falls(Unchalli Uttara Kannada District River Aghanashini 120 metres
Falls, also known as
Lushington Falls)
Shivanasamudra Falls Mandya District River Kaveri 90 metres
Languli Falls Uttara Kannada district River kali 62 metres
Gokak Falls Belagavi district River Ghataprabha 53 meters
Hogenakal Falls Chamrajnagar district River Kaveri 25 meters
Chunchanakatte Falls Mysore district River Kaveri 20 meters

15. Which of the following expression shows the formula of Gross fiscal deficit?
a) Gross Fiscal Deficit= Primary deficit+ Net borrowing from Abroad
b) Gross Fiscal Deficit = TotalExpenditure – Revenue Receipts
c) Gross Fiscal Deficit = Revenue Deficit+ Capital Expenditure
d) Gross Fiscal Deficit = Net Borrowing at home+ Borrowing from RBI+ Borrowing from abroad
Answer : D
 The gross fiscal deficit (GFD) is the excess of total expenditure including loans net of recovery over
revenue receipts (including external grants) and non-debt capital receipts. The net fiscal deficit is the
gross fiscal deficit less net lending of the Central government.
 As there are three sources of borrowing for the government, gross fiscal deficit is also estimated as the
sum total of borrowing from RBI, borrowing from abroad and net borrowing at home. Hence Option D
is correct
Additional Information
 Fiscal deficit takes place either due to revenue deficit or a major hike in capital expenditure. Capital
expenditure is incurred to create long-term assets such as factories, buildings and other development.
 A deficit is usually financed through borrowing from either the central bank of the country or raising
money from capital markets by issuing different instruments like treasury bills and bonds.
Measures of Government Deficit
Revenue Deficit:
 The revenue deficit refers to the excess of government’s revenue expenditure over revenue receipts
o Revenue deficit = Revenue expenditure – Revenue receipts
Fiscal Deficit:
 Fiscal deficit is the difference between the government’s total expenditure and its total receipts
excluding borrowing
o Gross fiscal deficit = Total expenditure – (Revenue receipts +Non-debt creating capital
receipt)
 Non-debt creating capital receipts are those receipts which are not borrowings and, therefore, do not
give rise to debt. Examples are recovery of loans and the proceeds from the sale of PSUs
 The fiscal deficit will have to be financed through borrowing. Thus, it indicates the total borrowing
17
MISSION SHOURYA PSI TEST SERIES -2024
requirements of the government from all sources. From the financing side
o Gross fiscal deficit = Net borrowing at home + Borrowing from RBI + Borrowing from
abroad
 Net borrowing at home includes that directly borrowed from the public through debt instruments (for
example, the various small savings schemes) and indirectly from commercial banks through Statutory
Liquidity Ratio (SLR). The gross fiscal deficit is a key variable in judging the financial health of the
public sector and the stability of the economy.
Primary Deficit:
 The goal of measuring primary deficit is to focus on present fiscal imbalances.
 To obtain an estimate of borrowing on account of current expenditures exceeding revenues, need to
calculate what has been called the primary deficit. It is simply the fiscal deficit minus the interest
payments
o Gross primary deficit = Gross fiscal deficit – Net interest liabilities
Net interest liabilities consist of interest payments minus interest receipts by the government on net domestic
lending.

16. When the central bank, increases the cash reserve ratio , it leads to
A. Fall in Money circulation
B. Low Demand for goods and services
C. Rise in purchasing power
D. Rise in Banks lending capacity
Choose the correct answer using the codes given below the answer
a) A,C and D
b) A and B
c) A, B and C
d) A , B and D
Answer: B
 Cash Reserve Ratio or CRR is a part of the RBI's monetary policy, which helps eliminate liquidity risk
and regulate money supply in the economy.
 If the CRR rate increases, banks' ability to issue loans decreases that leads to fall in money circulation.
Hence A is correct and D is incorrect
 When there is a fall in money circulation that effects through decrease in purchasing power results in
Low Demand for goods and services .Hence C is incorrect and B is correct
Additional Information
 Cash Reserve Ratio (CRR) is the minimum percentage of total deposits (ie. NDTL) that a commercial
bank is required to retain as cash reserves with the RBI.
 In simple words, CRR is the cash deposit a bank maintains with the RBI. CRR is governed by the
provisions of Section 42 of the Reserve Bank of India Act, 1934.
 It has to be in the form of Cash.
 It is applicable to all Scheduled commercial banks.
Difference between SLR and CRR
Cash Reserve Ratio Statutory Liquidity Ratio
The Cash Reserve Ratio (CRR) is the percentage of
Statutory Liquidity Ratio (SLR) is the minimum
deposits (NDTL) that a commercial bank is required
percentage of deposits (NDTL) that a commercial
to retain as cash reserves with the RBI. bank must keep with itself.
Reserves in the form of Cash Only Reserves in the form of Liquid cash, gold, or other
securities.
Maintained with Reserve Bank of India Maintained with Respective Banks
Helps meet short term liquidity requirements by Controls excess money flow in the economy.
trading excess securities.
18
MISSION SHOURYA PSI TEST SERIES -2024
Banks don't earn any interest on the CRR deposited Banks earns interest based on the portfolio of SLR
with RBI chosen.
Short-term liquidity management tool Long-term liquidity management tool.
How is CRR calculated?
CRR is calculated as the percentage of the bank’s NDTL that is net demand and time liabilities
The public and other banks can describe NDTL as the bank’s total demand and time liabilities (deposits)
minus deposits with other banks. The banks liabilities could take the form of
 Demand Liabilities such as current deposits , demand drafts and cash certificates
 Time liabilities such as FD’s , gold deposits and cash certificates
 Other demand and time liabilities such as deposits interest and dividends etc
Terminology
 Marginal Standing Facility (MSF): The rate at which banks can borrow overnight funds from RBI
against the approved government securities. Here, the borrowing limit is 2% of the banks’ Net Demand
and Time Liabilities (NDTL).
 Bank Rate: Higher rate (than the REPO rate) at which the RBI gives loans to other banks. A higher
bank rate would mean higher lending rates by banks. The RBI can raise the bank rate to check liquidity.
Here there is no 2% of NDTL limit.
 CRAR: Capital to Risk-Weighted Assets Ratio (CRAR): The ratio of a bank’s capital to its risk.
 REPO rate: Re Purchase Option (REPO): Rate at which the RBI gives loans to other banks.
 Reverse REPO rate: Rate at which the RBI borrows from other banks. It is lower than the REPO rate.

17. Human Development Index is a composite Index of which of the following ?


a) Nutritional status, Equality , quality of life and Per captia GDP
b) Poverty, life expectancy and educational attainment
c) Life expectancy, educational attainment and Per captia Income
d) Inflation ,Unemployment and Per capita GDP
Answer : C
 Human Development Index is a composite Index of Life expectancy, educational attainment and Per
captia Income. Hence Option C is correct
Additional Information
 The origins of the HDI are found in the annual Human Development Reports produced by the Human
Development Report Office of the United Nations Development Programme (UNDP).
 These were devised and launched by Pakistani economist Mahbub ul-Haq in 1990
Dimensions
 Life Expectancy Index: Calculated from Life expectancy at birth.
 Education Index: Calculated from Mean years of schooling and Expected years of schooling
 Income Index: Calculated from GNI per capita (PPP USD).
 India ranked at 132nd position in 2023.
Global Multidimensional Poverty Index (MPI)
 Released by the United Nations Development Programme (UNDP) and the Oxford Poverty and Human
Development Initiative (OPHI).
 Launched in 2010
Uses three standard dimensions:
 Health: child mortality, nutrition
 Education: years of schooling, enrollment
 Standard of Living and ten indicators: water, sanitation, electricity, cooking fuel, floor, assets
National Multidimensional Poverty Index

19
MISSION SHOURYA PSI TEST SERIES -2024
 Released by Niti Aayog and launched in 2021
 Three dimensions are represented by 12 indicators such as nutrition, child and adolescent mortality,
maternal health, years of schooling, school attendance, cooking fuel, sanitation, drinking water,
electricity, housing, assets, and bank accounts.
Global Hunger Index
 by Concern Worldwide and Welthungerhilfe
 100-point scale reflecting the severity of hunger - 0 is the best score (implies no hunger) and 100 is the
worst.
 four indicators: Undernourishment, Child Stunting, Child Wasting and Child Mortality
 India ranked 111th out of 125 countries, indicating a serious level of hunger.
Karnataka Human Development Report
 First Karnataka Human Development Report (KSHDR) was published in 1999
 Karnataka is the first state in the country to bring out report on Human Development
Karnataka Human Development Report 2023
 The top six districts where HDI increase is “significantly high” are Bagalkot, Ballari, Ramanagara,
Mandya, Yadgir and Koppal. Even in Chamarajanagar, the district considered to be the most backward,
HDI rose its ranking
 The 10 districts flagged for poverty are in north Karnataka -- Yadgir, Kalaburagi, Raichur, Koppal,
Ballari, Bidar, Gadag, Bagalkot, Vijayapura and Haveri.
 For the first time, the government has estimated HDI and MPI at the taluk level. Mangaluru taluk has
topped, leaving Bengaluru South second.
 Maternal mortality rate (MMR) is “comparatively high” in Dharwad, Chikballapur, Bidar, Hassan,
Haveri, Shivamogga, Tumakuru and Chamarajanagar

18. In the context of Indian economy, the term 'Twin Deficit refers to
a) Revenue deficit accompanied with Fiscal Deficit.
b) Occurrence of Current Account Deficit and Fiscal Deficit together.
c) Simultaneous Current Account Deficit and Capital Account Deficit
d) Simultaneous rise in the Non- Performing Assets of banks and debt- burden of the corporates
Answer: B
 Twin Deficit Problem: Current Account Deficit and Fiscal Deficit (also known as "budget deficit" is a
situation when a nation's expenditure exceeds its revenues) are together known as twin deficits and both
often reinforce each other, i.e., a high fiscal deficit leads to higher CAD and vice versa. Hence option B
is correct
Additional Information
 Fiscal Deficit: It is the gap between the government’s expenditure requirements and its receipts. This
equals the money the government needs to borrow during the year.
 Current Account Deficit (CAD): The current account measures the flow of goods, services, and
investments into and out of the country. It represents a country’s foreign transactions and, like the
capital account, is a component of a country’s Balance of Payments (BOP).
 Inflation is the rate of increase in prices over a given period of time. Inflation is typically a broad
measure, such as the overall increase in prices or the increase in the cost of living in a country.
 Deflation is an economic phenomenon characterized by a sustained decrease in the general price level
of goods and services within an economy. It is the opposite of inflation, where prices tend to rise over
time.
 Stagflation: It is described as a situation in the economy where the growth rate slows down, the level of
unemployment remains steadily high and yet the inflation or price level remains high at the same time.
 Black Swan Event: It can be characterised by a combination of all adverse shocks experienced in the
history coming together, leading to a perfect storm.
 Disinflation is a temporary slowing of the pace of price inflation. The term is used to describe

20
MISSION SHOURYA PSI TEST SERIES -2024
occasions when the inflation rate has reduced marginally over the short term. Unlike inflation and
deflation, which refer to the direction of prices, disinflation refers to the rate of change in the rate of
inflation.
 NPA refers to a classification for loans or advances that are in default or are in arrears on scheduled
payments of principal or interest.Debt is classified as non-performing, when the loan payments have not
been made for a minimum period of 90 days.
 Gross non-performing assets are the sum of all the loans that have been defaulted by the individuals
who have acquired loans from the financial institution.
 Net non-performing assets are the amount that is realised after provision amount has been deducted
from the gross non-performing assets.
 Balance of Payments (BOP): It is the difference in total value between payments into and out of a
country over a period.
 Balance of Trade (BOT): It is the difference between a country’s imports and exports for a time
period. The BOT is a part of the BOP.

19. GST is /an_______?


a) Destination based consumption tax
b) Origin- based production tax
c) Destinations based sales tax on transaction sales
d) Origin based tax on transaction
Answer : A
 The Goods and Services Tax (GST) is a destination-based, value-added tax that was introduced in India
on July 1, 2017. Hence Option A is correct
Additional Information
 The recommendation for the introduction of Goods and Services Tax (GST) in India was made by the
Kelkar Task Force on indirect taxes in 2003
 GST Bill was first introduced in 2014 as The Constitution (122nd Amendment) Bill.
 This got an approval in 2016 and was renumbered in the statute by Rajya Sabha as The Constitution
(101st Amendment) Act, 2016
 Article 246A – States have power to tax goods and services.
 Assam is 1st State to pass the GST bill.
GST Council
 It is a constitutional body
 Article 279A - GST Council to be formed by the President to administer & govern GST.
 It's Chairman is Union Finance Minister of India with ministers nominated by the state governments as
its members.
 The council is devised in such a way that the centre will have 1/3rd voting power and the states have
2/3rd.
 The decisions are taken by 3/4th majority
 Tax slabs are decided as 0%, 5%, 12%, 18%, and 28% along with categories of exempted and zero-rated
goods for different types of goods and services.
GSTN
 GSTN is registered as a not-for-profit company under the Companies Act.
 It has been formed to set up and operate the information technology backbone of the GST.
 GSTN had awarded Infosys Ltd the contract to develop the hardware and software for GST.
E-Way Bill
 An e-way bill, short form for electronic way bill, is a permit needed for inter-state and intra-state
transportation of goods worth more than Rs. 50,000.
 It contains details of the goods, the consignor, the recipient and the transporter.
21
MISSION SHOURYA PSI TEST SERIES -2024
 It can be electronically generated through the Goods and Services Tax Network (GSTN).
e-Invoice
 e-Invoice is a system in which B2B invoices are authenticated electronically by GSTN for further use
on the common GST portal.
 Under the electronic invoicing system, an identification number will be issued against every invoice by
the Invoice Registration Portal (IRP) to be managed by the GST Network (GSTN).

20. Which of the following statements is correct regarding diamond and graphite?
a) Diamond and graphite have similar physical and chemical properties
b) Diamond is hard but graphite is smooth and slippery
c) Diamond and graphite are both nonconductors of electricity
d) Both diamond and graphite have similar structures
Answer: B
 Diamond is hard, as the carbon atoms in diamond are bonded in a stronger tetrahedron pattern but
graphite is soft and slippery because the carbon atoms in graphite are bonded in layers with only weak
vanderwall force holding the layers together. Hence Option B is correct
Additional Information
 Both the physical and chemical properties of diamond and graphite are different. An allotrope is another
form of the same element having different physical properties but the same chemical properties. For
example, Graphite and diamond are two different allotropes of carbon
Diamond Graphite
It has three dimensional network structure It has two dimensional network structure
Each carbon atom is sp3 hybridized Each carbon atom is sp2 hybridized
Each carbon atom in a diamond is linked to four Each carbon atom in a diamond is linked to three
other carbon atoms other carbon atoms
They are poor conductor of electricity due to the They are good conductor of electricity due to the
absence of free electrons absence of free electrons
It is the hardest known natural substance It is soft and slippery

21. Match the following


Blood vessels Function
A. Pulmonary 1. Carries blood from
artery lungs to heart
B. capillaries 2. Carries blood to
heart
C. Coronery 3. Connection
artery between arteries
and veins
D. veins 4. Carries blood from
heart to lungs
Codes
a) A-4, B-3, C-2 ,D-1
b) A-4, B-2, C-3 ,D-1
c) A-1, B-3, C-2 ,D-4
d) A-1, B-2, C-3 ,D-4
Answer : A
 An artery is a blood vessel that transports oxygen-rich or oxygenated blood from the heart to the other
parts of the body
o Pulmonary artery which carries deoxygenated blood from the heart to the lungs
o Coronary arteries supply blood to the heart muscle. It wrap around the outside of the heart
22
MISSION SHOURYA PSI TEST SERIES -2024
 A vein is a blood vessel that transports deoxygenated blood from the body parts to the heart
o Pulmonary vein which transports oxygenated blood from lungs to the heart. It works in opposite to
Pulmonary artery
o The veins connect to form larger veins . Each lung has two pulmonary veins that carry blood to the
top left chamber of the heart known as atrium
 Capillaries are delicate blood vessels that exist throughout your body. They transport blood, nutrients
and oxygen to cells in your organs and body systems. Capillaries are the smallest blood vessels in your
vascular system.
 The lungs are the respiratory organs in humans which helps in the exchange of gases.

22. Which of the following is NOT correctly matched?


a) Desalination – Sea water
b) Reverse Osmosis- Potable water
c) Denaturation – Proteins
d) Pasteurization – Tea
Answer: D
Pasteurization:
 Pasteurization (or pasteurisation) is the process by which heat is applied to food and beverages to kill
pathogens and extend shelf life. Typically, the heat is below the boiling point of water (100 °C or 212
°F).
 Ex: whole milk, skim milk, standardized milk and various types of cream.
 The process includes heating milk to a high temperature and then quickly cooling it before bottling or
packaging. It helps to keep it fresh for a longer time. The heating of milk is done up to (and no more
than 25 seconds).It process is also referred to as the ‘High-Temperature Short Time’ (HTST) process.
Hence Option D is Incorrect
Additional Information
 Desalination is a process used to separate dissolved salts and other minerals from water.
 Reverse osmosis is a process by which a solvent passes through a porous membrane in the direction
reverse to that for natural osmosis when subjected to a hydrostatic pressure greater than the osmotic
pressure.
o Ex: Industries that use RO water include pharmaceutical, boiler feed water, food and beverage,
metal finishing and semiconductor manufacturing
 Denaturation is a process of modifying the molecular structure of a protein by breaking many of the
weak bonds within a protein molecule that is responsible for the highly ordered structure of the protein
in its natural state.
o Ex: denaturing that occur in everyday life include cooking eggs and meat, digestion, and the use of
alcohol for disinfection.

23. Consider the following statements regarding Vikram-S


A. Its maiden flight lifted on 18 November 2022
B. The first mission of the rocket launch has been designated as 'Prarambh'
C. It is developed by ISRO
Choose the correct answer using the codes given below the answer
a) A Only
b) A and B
c) C Only
d) All of the above
Answer : B
Vikram-S rocket is developed by Skyroot Aerospace and the launch was assisted by ISRO and IN-SPACe
(Indian National Space Promotion and Authorisation Centre). Hence statement C is incorrect

23
MISSION SHOURYA PSI TEST SERIES -2024
 Vikram-S is India's first privately built rocket. Its maiden flight lifted on 18 November 2022 at 11 A.M.
from Sounding Rocket Complex, Sriharikota.
 The first mission of the rocket launch has been designated as 'Prarambh', meaning beginning.
 As a tribute the rocket has been named after Vikram Sarabhai, who is considered as the father of India's
space programme.
Additional Information
Here is a list of countries which have private space agencies capable of launching spacecrafts, satellites and
probes into space:
UNITED STATES
 Elon Musk’s SpaceX has achieved many firsts as a private company in the space industry.
 SpaceX was the first to send astronauts to orbit and to the International Space Station and also the first
private company to send a spacecraft to the International Space Station (ISS).
 It is also known for recovering and reusing its spacecraft and also for mastering the first vertical take-off
and vertical propulsive landing for an orbital rocket booster.
 Along with SpaceX, the US also has Amazon CEO Jeff Bezos’ founded Blue Origin, which has sent a
crewed mission to space and also plans to launch the Blue Moon lander by 2024.
 Boeing and the Sierra Nevada Corporation (SNC) Space Systems also have the ability to send cargo to
space. Boeing’s CST-100 Starliner has already attempted to reach the ISS but it failed due to technical
difficulties earlier in 2019. Both Boeing and SNC will be able to launch their CST-100 Starliner and
Dream Chaser Space System respectively next year.
 The Orbital Science Corporation which has developed the Cygnus is also another private company
which is capable of launching rockets and satellites into space. It is now a part of the Northrop
Grumman Corporation.
RUSSIA
 There are several other companies like Cosmocourse, S7Space, LinIndustrial and Sputnix who are
planning to launch crewed missions as well as small satellites to space.
 The United Launch Alliance – a joint venture between Lockheed Martin and Boeing – is also capable of
launching rockets into space.
 Richard Branson’s Virgin Atlantic owned Virgin Galactic is also capable of sending crewed missions
and is planning to launch small satellites into space.
JAPAN
 Interstellar Technologies and ispace are two companies that are planning to launch not only payloads
but also lunar missions within 2023. The MOMO sounding rocket also launched a payload into space.
FRANCE AND THE EUROPEAN UNION
 France’s ArianeSpace is the among the world’s first commercial launch service providers and boasts the
Vega, Soyuz and Ariane 5 rockets which have launched several hundred satellites and other payloads
into space.
 Its subsidiaries are France, Belgium, Germany, Italy, Netherlands, Norway, Spain, Sweden and
Switzerland.
CHINA
 China’s rocket developer Galactic Energy, a Beijing-based aerospace company launched commercial
rocket CERES-1 Y2.
 It first launched a small satellite into space in 2020.
 i-Space is another private Chinese company, which was the first to launch a satellite. In 2019, iSpace
launched the Hyperbola-1 and reached low Earth orbit on its first-ever flight.

24
MISSION SHOURYA PSI TEST SERIES -2024
24. Hope Probe mission lanched by which of the following countries ?
a) UAE
b) United States
c) Russia
d) European Union
Answer: A
 UAE’s first-ever interplanetary Hope Probe mission has successfully entered orbit around Mars. The
mission was launched from Tanegashima Space Centre in Japan.
 Mission’s aim: The mission will collect data on Martian climate dynamics and help scientists
understand why Mars atmosphere is decaying into space.
Additional Information
TIANWEN-1
 It is the first mission to Mars from China.
 It carries an orbiter, a lander, and a rover.
2001 Mars Odyssey
 It is a robotic spacecraft orbiting the planet Mars. The project was developed by NASA, and contracted
out to Lockheed Martin
The Mars Reconnaissance Orbiter
 It is a spacecraft designed to search for the existence of water on Mars and provide support for missions
to Mars, as part of NASA's Mars Exploration Program. It was launched from Cape Canaveral on August
12, 2005,
PERSEVERANCE MISSION
 NASA’s 4th generation Mars Rover. It is the most advanced, most expensive & most sophisticated
mobile laboratory sent to Mars. It is different from previous missions as it can drilling & collect core
samples of most promising rocks & soils.
MAVEN
 It NASA spacecraft orbiting Mars to study the loss of that planet's atmospheric gases to space,
providing insight into the history of the planet's climate and water.
Mars Express
 It is a space exploration mission being conducted by the European Space Agency (ESA).
 The Mars Express mission is exploring the planet Mars, and is the first planetary mission attempted by
the agency
MARS InSight Mission
 The Interior Exploration using Seismic Investigations, Geodesy and Heat Transport (InSight) mission
was a robotic lander designed to study the deep interior of the planet Mars.
 It was manufactured by Lockheed Martin Space, was managed by NASA's Jet Propulsion Laboratory
(JPL) and two of its three scientific instruments were built by European agencies.
Curiosity
 It is a car-sized Mars rover exploring Gale crater and Mount Sharp on Mars as part of NASA's Mars
Science Laboratory mission.
Mangalyaan
 It was launched from Satish Dhawan Space Centre in Andhra Pradesh by ISRO in 2013. It was launched
on board a PSLV C25 rocket with aim of studying Martian surface and mineral composition as well as
scan its atmosphere for methane (an indicator of life on Mars).

25
MISSION SHOURYA PSI TEST SERIES -2024
The ExoMars Trace Gas Orbiter
 It is a collaborative project between the European Space Agency and the Russian Roscosmos agency
that sent an atmospheric research orbiter and the Schiaparelli demonstration lander to Mars in 2016 as
part of the European-led ExoMars programme

25. The hierarchical level of organization in ecology is ?


a) Organism, Community, Population , Biome, Ecosystem , Ecosphere , Landscape
b) Community, Population, Organism, Ecosphere , Landscape , Biome, Ecosystem
c) Organism, Population , Ecosystem , Community, Landscape , Ecosphere ,Biome,
d) Organism, Population ,Community, Ecosystem , Biome, Landscape , Ecosphere
Answer: D
 Ecological hierarchy refers to the synergy of organisms with their environment and leads to the
formation of a grouping of organisms.

 Organism or single individual: An organism is a single individual or being.


 Population: It is a group of individuals of a plant or an animal species inhabiting a given area.
 Biological community: Assemblage of populations of plants, animals, microbes and all other life
forms living in a particular area and interacting with each other for fulfilment of needs.
 Ecosystem: Composed of biological community, integrated with its physical environment through
exchange of gases, energy & recycling of nutrients. Ecosystem involves interactions between living and
non-living worlds or biotic and abiotic worlds.
 Biome: A large regional unit characterized by major vegetation types and associated fauna/animal life,
found in specific climatic zone. Ex. Tropical rain forests of Western Ghats form a typical biome with
characterized vegetation of mahogany, ebony, rosewood etc. accompanied with animal life of Malabar
civet, Nilgiri Tahr etc. and climatic conditions of high rainfall, high humidity and higher temperatures.
 Landscape: A unit of land with a natural boundary having a mosaic of patches representing different
ecosystems. Ex. a view of river, its valley & grasslands nearby from a mountain top with three different
ecosystems in one picture.
 Ecosphere: On a global scale, all the earth’s terrestrial biomes & aquatic systems constitutes the
26
MISSION SHOURYA PSI TEST SERIES -2024
biosphere. It includes lower atmosphere, land, oceans & rivers etc. where living organisms can be
found. So, biosphere is the biologically inhabited part of earth along with its physical environment.

26. Consider the following:


1. Carbon monoxide
2. Nitrogen oxide
3. Ozone
4. Sulphur dioxide
Excess of which of the above in the environment is/are cause(s) of acid rain?
a) 1, 2 and 3
b) 2 and 4 only
c) 4 only
d) 1, 3 and 4
Answer : B
 The term “acid rain” refers to the precipitation of acid in the form of rain. Acid Rain occurs when
atmospheric pollutants such as oxides of nitrogen and sulphur, interact with rainwater and fall as a
result. Hence option B is correct
 Acid rain is the rain that has been acidified, with a pH less than 5.6.
Additional Information
 Eutrophication is the process in which a water body becomes overly enriched with nutrients, leading to
the plentiful growth of simple plant life. The excessive growth (or bloom) of algae and plankton in a
water body are indicators of this process.
 Ocean acidification is the ongoing decrease in pH of the Earth's ocean water, caused by the absorption
of carbon dioxide (CO2) from the atmosphere. When CO2 dissolves in seawater, it forms carbonic acid,
which increases the acidity of the water.

27. Which of the following is/ are examples of green house gases?
a) Nitric Acid
b) Nitrogen Dioxide
c) Nitrous Oxide
d) All of the above
Answer: C
 Nitric acid is used as a water treatment to lower the pH, this chemical also serves as an excellent source
of nitrogen which is excellent for the soil as it contributes to healthy plant growth and fuller crops.
Hence Option A is incorrect
 Nitrogen dioxide, or NO2, is a gaseous air pollutant composed of nitrogen and oxygen. The main source
of nitrogen dioxide resulting from human activities is the combustion of fossil fuels (coal, gas and oil)
especially fuel used in cars. Hence Option B is incorrect
 Nitrous oxide enhances the greenhouse effect just as carbon dioxide does by capturing reradiated
infrared radiation from the Earth's surface and subsequently warming the troposphere (lower
atmosphere). Hence Option C is Correct
Additional Information
Carbon dioxide (CO2)  Burning of fossil fuels, deforestation
Methane (CH4)  Growing paddy, excreta of cattle and other livestock,
termites, burning of fossil fuel, wood, landfills, wetlands,

27
MISSION SHOURYA PSI TEST SERIES -2024
fertilizer factories.
Nitrous oxides (N2O)  Burning of fossil fuels, fertilizers; burning of wood and crop
residue.
Per fluorocarbons (PFCs)  Produced as a by-product in aluminum production and
manufacturing of semiconductors.
Hydro fluorocarbons (HFCs)  Used as refrigerants, aerosol propellants, solvents and fire
retardants.
Sulphur hexafluoride (SF6)  Used as tracer gas for leak detection, used in electrical
transmission equipment

28. According to Tiger census data 2022, Karnataka’s tiger population is


a) 563
b) 564
c) 565
d) 566
Answer: A
 The number of tigers in India has increased from 2,967 in 2018 to 3,682 in 2022, an annual rise of 6 per
cent, according to the latest government data released on the International Tiger Day
 India has become home to approximately 75 per cent of the world’s tiger population
 Madhya Pradesh has the maximum number (785) of tigers in the country, followed by Karnataka (563),
Uttarakhand (560), and Maharashtra (444). Hence Option A is correct
Additional Information
 Arunachal Pradesh, Odisha, Telangana, Chhattisgarh and Jharkhand have recorded a dip in the number
 The number of tigers “within the tiger reserve” is highest in Corbett (260), followed by Bandipur (150),
Nagarhole (141), Bandhavgarh (135), Dudhwa (135), Mudumalai(114), Kanha (105), Kaziranga (104),
Sundarbans (100), Tadoba (97), Sathyamangalam (85), and Pench-MP (77).
 The Western Ghats witnessed a decrease in tiger occupancy, except for a few areas like Kali (Anshi
Dandeli)
Tiger Reserve in Karnataka
 Bandipur Tiger Reserve
 Bhadra Tiger Reserve
 Biligiri Ranganatha Temple Tiger Reserve Karnataka
 Dandeli-Anshi (Kali) Tiger Reserve
 Nagarhole Tiger reserve
National Park
 Bandipur Tiger Reserve
 Nagarhole Tiger Reserve
 Bannerghatta National Park
 Kudremukh National Park
 Anshi National Park
Note : There are 5 Tiger reserve and 5 National Park in Karnataka

28
MISSION SHOURYA PSI TEST SERIES -2024
29. Match List I (Environment legislation) with List II (Year ) and select the correct answer using the
codes given below the list.
Environment Year
legislation
A. The Water (Prevention 1. 2010
and Control of
Pollution) Act
B. The Air (prevention 2. 1972
and control of
pollution) act
C. The National Green 3. 1974
Tribunal Act
D. The Wildlife 4. 1982
(Protection) Act
5. 1981
Codes
a) A-5, B-4,C-1,D-2
b) A-3, B-5,C-2, D-1
c) A-5, B -4,C-2,D-1
d) A-3, B-5,C-1,D-2
Answer: D

Environment legislation Year


The Indian Forest Act 1927
The Wildlife (Protection) Act 1972
The Water (Prevention and Control of Pollution) 1974
Act
The Forest Conservation Act 1980
National Forest Policy 1988
The Air (prevention and control of pollution) act 1981
The Environment (Protection) Act 1986
The ozone-depleting substances (regulation and 2000
control) rules.
The Municipal Solid Wastes (Management and 2000
Handling) Rules
The energy conservation act 2001
Biological diversity act 2002
Scheduled Tribes and Other Traditional Forest 2006
Dwellers (Recognition of Forest Rights) Act, (FRA)
The National Green Tribunal Act, 2010
Compensatory Afforestation Fund Act 2016
Solid Waste Management Rules 2016
Plastic Waste Management Rules Came in forces on 2016 and Amendment for is
Rules, 2018
Bio-Medical Waste Management Rules Came in forces on 2016 and Amendment for is
Rules, 2018
E-Waste Management Rules Came in forces on 2016 and Amendment for is
Rules, 2018
Coastal Regulation zone notification 2018

29
MISSION SHOURYA PSI TEST SERIES -2024
30. India’s rank in Global Hunger Index is
a) 110th
b) 111th
c) 113th
d) 112th
Answer : B
 India ranks 111 out of a total of 125 countries in the Global Hunger Index (GHI) 2023
 The country came after neighbouring countries Pakistan (102nd), Bangladesh (81st), Nepal (69th) and
Sri Lanka (60th)
 As per the index, India also has the highest child wasting rate in the world
 Belarus, Bosnia & Herzegovina, Chile, China are among the top ranked countries (i.e., low level of
hunger) and Yemen, Madagascar, Central African Republic are the bottom.

31. National Turmeric Board is established in


a) Karnataka
b) Maharashtra
c) Telangana
d) Tamil Nadu
Answer: C
 Government of India notified the constitution of the National Turmeric Board in Telangana under
Ministry of Commerce & Industry of the Union Government
Composition
 The Board shall have a Chairperson, who will be appointed by the Central Government.
 It will have members from the Ministry of AYUSH, Departments of Pharmaceuticals, Agriculture &
Farmers Welfare, Commerce & Industry of the Union Government, senior State Government
representatives from three states (on rotation basis).
 Select national/state institutions involved in research, representatives of turmeric farmers and exporters.
 It will have a Secretary to be appointed by the Department of Commerce.
 The Ministry of Commerce will be the nodal department providing funds and infrastructure for the
NTB, which will look into increased demand, usage, production, research, market linkage, exports and
so on.
Turmeric (Curcuma longa)
 It is a perennial herbaceous plant belongs to the ginger family (Zingiberaceae).
Climatic conditions required
 Soil and climate: A friable well drained red loamy soil in wet or garden lands under tropical conditions
is ideal.
 Rain: It can be grown in regions receiving an annual rainfall of 1500 mm.
 Temperature: range of 20 to 35 °C.
 India is the largest producer, consumer and exporter of turmeric in the world.
 The largest producing states of Turmeric are Maharashtra, Telangana, Karnataka and Tamil Nadu
Department of Commerce, Ministry of Commerce and Industry
Tea Board of India Kolkata
Coffee Board of India Bengaluru
Rubber Board Kottayam
Spices Board of India Kochi
Tobacco Board Guntur

30
MISSION SHOURYA PSI TEST SERIES -2024
Ministry of AYUSH
National medicinal plants board New Delhi
Ministry of Agriculture and Farmers' welfare
National Horticulture Board Gurugram
Coconut Development Board Kochi
Ministry of Textiles
Central Silk Board Bengaluru
Jute Board Kolkata

32. Who among the following won Vi Kru Gokak Award 2023?
a) Devanur Mahadeva
b) Chandrashekhara Kambara
c) Chiranjiv Singh
d) Lakshmeesha tolpadi
Answer: C
 Vinayaka Gokak Vangmaya Trust has chosen former IAS officer Chiranjiv Singh for the Vi Kru Gokak
Award 2023

33. Consider the following statement regarding Ksheera Bhagya scheme


A. The scheme was launched in the year 2013
B. Milk is served to children studying in Classes 1-10 in government schools five days a week,
whereas the same is provided to beneficiaries at anganwadis six days a week.
Choose the correct answer using the codes given below the answer
a) A only
b) B only
c) Both of them
d) None of the above
Answer: c) Both of them
 Recently government marked 10 years of the launch of the Ksheera Bhagya scheme.
 Scheme was launched by KMF in association with lower an higher education department , women and
child department
 The scheme was started during his previous tenure as chief minister in 2013.
 Milk is served to children studying in Classes 1-10 in government schools five days a week, whereas the
same is provided to beneficiaries at anganwadis six days a week
 Aim is to provide nutritious food with protein and fat which is essential for healthy growth and all-
round development of School Children as well as Anganwadi in the State.
 Beneficiaries are 1st to 10th standard in government and government aided schools in all over
Karnataka and Children from 6 months to 6 years in anganwadi
Eligibility for Ksheera Bhagya Scheme:
 Student should be resident of Karnataka state
 Students from Class 1st to 10th studying in government and government-aided schools and Anganwadi
children’s will get benefits under this scheme
 The milk is provided for 5 days in a week
 Only government and government-aided schools and aganwadis are eligible for this scheme

31
MISSION SHOURYA PSI TEST SERIES -2024

34. ‘A-HELP' Programme is launched by


a) Ministry of Fisheries, Animal Husbandry & Dairying
b) Ministry of Commerce and Industry
c) Ministry of Education
d) Ministry of Youth Affairs & Sports
Answer: A
 It is an initiative of the Union Ministry of Fisheries, Animal Husbandry & Dairying and the Ministry of
Rural Development.
 ‘A-HELP’ are community-based women activists, who assist veterinarians in local departmental
activities help livestock farmers to take loans for entrepreneurship development, fill out applications,
mark the ear tagging of animals and register them in the INAF portal and help with insurance etc.
 They assist in implementing various schemes and in providing information to the farmers at the
grassroots level.
 They will give vital contributions to preventing various infectious diseases of animals, artificial
insemination under the Rashtriya Gokul Mission (RGM), tagging animals and animal insurance.
Rashtriya Gokul Mission
 It is being implemented for the development and conservation of indigenous bovine breeds since
December 2014.
 It is continued under the umbrella scheme Rashtriya Pashudhan Vikas Yojana from 2021 to 2026.
 Objectives: To enhance the productivity of bovines and increase milk production in a sustainable
manner using advanced technologies

35. The famous 'Anguttara Nikaya' is a part of which of the following ancient religious texts?
a) Sutta Pitaka
b) Abhidhamma pitaka
c) Milinda Panha
d) Vinaya Pitaka.
Ans: A.Sutta Pitaka
Buddhist Literature
 During his lifetime the Buddha taught not in Vedic Sanskrit, which had become tough to the people, but
in his own Indian dialect, he also encouraged his monks to propagate his teachings in the vernacular.
After his death, the Buddhist canon was formulated and transmitted by oral tradition, and it was written
down in several versions in the 2d and 1st cent. B.C. Its main divisions, called Pitakas.
 Tripitaka, also known as Pali Canon in English, is a traditional term used for Buddhist scriptures. The
three pitakas are Sutta Pitaka, Abhidhamma Pitaka and Vinaya Pitaka.
 It has around 10 thousand sutras related to Buddha and his close associates.
 It also deals with the first Buddhist council, held shortly after Buddha’s death.
Sutta Pitaka is divided under following sections:
 •Anguttara Nikaya which comprises the numerical.

• Digha Nikaya, which comprises the long discourses.

• Khuddaka Nikaya which comprises the minor collection.

• Majjhima Nikaya, which comprises the middle length.

• Samyutta Nikaya which comprises the connected discourses of Buddha.

32
MISSION SHOURYA PSI TEST SERIES -2024
Vinaya Pitaka
 Also known as book of discipline, it deals with the monastic rules for monks and nuns. It is further
divided into three books namely Suttavibhanga, Khandhaka and Parivara.
Abhidhammapitaka
 It comprises the philosophy and doctrine of Buddhism. It is divided into seven books namely
Dhammasangani, Dhatukatha, Kathavatthu, Patthana, Puggalapannatui, Vibhanga and Yamaka.
Jatakas
 It comprises the stories of previous births of Buddha in the form of poems.
Milinda Panha
 It contains the dialogue between Buddhist monk Nagasena and Indo-Greek king Meander.
Dipavamsa
 Dipavamsa means Chronicle of Island. In fact, it is the oldest historical record of Sri Lanka. It is
considered one of the most important works in Pali Literature.
Mahavamsa
 Its literal meaning is Great Chronicle. It is the most important Pali epic poem. Its tone is historical and it
describes the kings of Sri Lanka. The book is one of the longest historical account.

36. With reference to Revolutionary Extremism, consider the following pairs:


Activity Associated Person
1. Chittagong Armoury Raid A. Anant Singh
2. Kakori Robbery B. Rajendra Lahiri
3. Bolshevik Conspiracy case C. Ganesh Gosh
Which of the pairs given above is/are correctly matched
a) 1 and 2 only
b) 2 and 3 only
c) 1 and 3 only
d)1, 2 and 3
Ans: (a) 1 and 2 only
Chittagong Armoury Raid – Features
 The Chittagong armory raid, also known as the Chittagong uprising, was an attempt on 18 April 1930
by armed Indian independence fighters led by Surya Sen to raid the armory of police and auxiliary
forces from the Chittagong armory in the Bengal Presidency of British India (now in Bangladesh).
 They were inspired by the Easter Rising in Ireland in 1916.
 However, the Communists in Soviet Russia had a greater ideological influence on them. Many of these
raiders later joined the Communist Party.
 Ganesh Ghosh, Lokenath Bal, Ambika Chakraborty, Harigopal Bal (Tegra), Ananta Singh, Anand
Prasad Gupta, Tripura Sen, Bidhubhusan Bhattacharya, Pritilata Waddedar, Kalpana Dutta, Himangshu
Sen, Binod Bihari Chowdhury, Subodh Roy, and Monoranjan Bhattacharya were among those
present.Hence 1st pair is correctly matched
Kakori Robbery - Features
 The Kakori Conspiracy (also known as the Kakori train robbery) was a train robbery that occurred near
Lucknow. Hindustan Republican Association (HRA) organised the robbery.
 On August 9, 1925, the Number 8 Down Train travelling from Shahjahanpur to Lucknow was
approaching the town of Kakori when a revolutionary pulled the emergency chain to stop the train and

33
MISSION SHOURYA PSI TEST SERIES -2024
overpowered the guard.
 The guard cabin was the target, as it was transporting money collected from various railway stations to
be deposited in Lucknow. Only those bags (containing more than 100,000 rupees) were looted by
revolutionaries, who fled to Lucknow.
 Despite the fact that no passengers were targeted by the revolutionaries, one passenger, Ahmed Ali, was
killed in the crossfire between the guards and the revolutionaries. As a result, the case was classified as
manslaughter.
 Ram Prasad Bismil, Ashfaqulla Khan, Rajendra Lahiri, Chandrashekhar Azad, Swaran Singh,
Sachindra Bakshi, Keshab Chakravarty, Manmathnath Gupta, Mukundi Lal, Banwari Lal, Kundan Lal,
and Pranawesh Mukherjee carried out the robbery plan.
 The goals of this robbery were to:
o The money stolen from the British administration will be used to fund the HRA.
o Obtain public attention by promoting a favourable image of the HRA among Indians.
 Following the incident, the British administration launched a massive manhunt and arrested a number of
revolutionaries who were members or associates of the HRA.
 Their leader, Ram Prasad Bismil, was arrested in Shahjahanpur on October 26, 1925, and Ashfaqullah
Khan was arrested in Delhi on December 7, 1926. Hence 2nd pair is correctly matched
Kanpur Bolshevik Conspiracy Case (1924 AD)
 In this case, newly emerged communists of India were execrated by the British Government. M N Roy,
Muzaffar Ahamed, S A Dange, Shaukat Usmani, Nalini Gupta, Singaravelu Chettiar, Ghulam Hussain
were caught by the Government and were trailed for conspiring against the Government. They were
charged:
 “to deprive the King-Emperor of his sovereignty of British India, by complete separation of India from
imperialistic Britain by a violent revolution.”
 This case was not a people movement but a British movement to sack the upcoming communist leaders
of the time.Hence 3rd pair is not correctly matched

37. Consider the following pairs


Mountain Continent
Range
1. Atlas A. Africa
2. Appalachians B. South America
3. Urals C. Asia
4. Vinson Massif D. Antarctica
5. Andes E. North America
Which of the pairs given above are correctly matched
a) 1, 2 and 3 only
b) 1,3 and 4 only
c) 2 and 5 only
d)1,2,3,4 and 5
Ans: (b) 1,3 and 4 only
Mountain Ranges
Sr. Mountain Important/Highest Peaks Location Description
No. Range
1 Rocky Mt. Elbert (highest peak in North America It is one of the longest fold
Mountains the Rockies) mountains in the world and
extends from Canada to

34
MISSION SHOURYA PSI TEST SERIES -2024
Western US (New Mexico
State)
2 Appalachian Mt. Mitchell, North North America It is a fold mountain with
Mountains Carolina, US (highest peak rich in mineral resources
of Appalachian Mountains)
3 Alps Mont Blanc (French –Italian Europe It is a folded mountain and
border) source for rivers like
Danube, Rhine, etc.
4 Sierra Nevada Mt. Whitney California, USA Habitat for many Red Indian
tribes
5 Alaska Range Mt. McKinley North America Mt. McKinley highest peak
in North America
6 Altai Belukha mountain Central Asia Young folded mountain
Mountains which extends from
Kazakhstan to northern
China.
7 Andes Mt. Aconcagua South America Longest mountain chain in
Mountains the world
8 Atlas Mt. Toubkal Northwestern Young fold mountain
Mountains Africa spreading over Morocco and
Tunisia.
9 Drakensberg Mt. Lesotho South Africa Young folded mountain
Mountains
10. Caucasus Mt. Elbrus Europe Located between the Black
Mountain Sea and the Caspian Sea
11. Ural Mountains Mt. Narodnaya Russia This mountain range act as a
boundary between Europe
and Asia.
12. Hindukush Mt. Trich Mir Pakistan and Folded mountain with
Mountains Afghanistan rugged topography which
makes it difficult for
transportation.
13. Himalayas Mt. Everest Asia Young fold mountains in
Asia which separates Indian
sub-continent from Asian
plains
14. Arakan Yoma Mt. Kennedy peak Myanmar It extends from north to
south direction. Shifting
cultivation is practised.
15. Kunlun Mt. Muztag North of Tibetan It is one of the young folded
Mountains plateau and mountains.
western China
16. Vosges Mt. Grand Ballon Eastern France, Famous for the cultivation of
Europe grapes and manufacture of
wines.
17. Great Dividing Mt. Kosciuszko Australia This range is the source for
Range the rivers Darling and
35
MISSION SHOURYA PSI TEST SERIES -2024
Murray.

pair 2 and 5 are incorrectly matched

38. Which of the following pairs of the Constitution is/are correctly matched with the subject matter
they deal with?
Schedule subject
1. 4th Schedule A. Allocation of seats in
the Council of
States(Rajya Sabha).
2. 6th schedule B. The Administration
and Control of
Scheduled Areas and
Scheduled Tribes in
all the states of India
3. 8th schedule C. List of recognized
Languages
Choose Correct Option From Below
a) 1 and 2 only
b) 3 only
c) 1 and 3 only
d) 1,2 and 3
Ans: (c) 1 and 3 only
About Sixth Schedule of the Constitution
 It provides for the administration of tribal areas in Assam, Meghalaya, Tripura and Mizoram to
safeguard the rights of the tribal population in these states.Hence 2nd pair is incorrect.
36
MISSION SHOURYA PSI TEST SERIES -2024
 This special provision is provided under Article 244(2) and Article 275(1) of the Constitution.
 It seeks to safeguard the rights of the tribal population through the formation of Autonomous District
Councils (ADC).
 ADCs are bodies representing a district to which the Constitution has given varying degrees of
autonomy within the state legislature.
List of Schedules of Indian Constitution
Schedules in Indian Constitution
Schedules Features of Schedules
First Schedule of Indian  It contains the name of States and Union Territories
Constitution  Territorial Jurisdiction of states is also included
Second Schedule of Indian The provisions in relation to allowances, privileges, emoluments of:
Constitution  President of India
 Governors of Indian States
 Speaker of Lok Sabha & Deputy Speaker of Lok Sabha
 Chairman of Rajya Sabha & Deputy Chairman of Rajya Sabha
 Speaker and Deputy Speaker of Legislative Assemblies of Indian States
 Chairman and Deputy Chairman of Legislative Councils of the Indian
States
 Supreme Court Judges
 High Court Judges
 Comptroller & Auditor General of India (CAG)
Third Schedule It contains the forms of oath and affirmation for:
 Union Ministers of India
 Parliament Election Candidates
 Members of Parliament (MPs)
 Supreme Court Judges
 Comptroller and Auditor General
 State Ministers
 State Legislature Elections’ Candidates
 State Legislature Members
 High Court Judges
Fourth Schedule  It contains the provisions in relation to the allocation of seats for States
and Union Territories in the Rajya Sabha
Fifth Schedule  It contains provisions in relation to the administration and control of
scheduled areas and scheduled tribes
Sixth Schedule  It contains provisions in relation to the administration of tribal areas in
the states of Assam, Meghalaya, Tripura and Mizoram
Seventh Schedule This schedule deals with the three legislative lists:
 Union
 State
 Concurrent
Eighth Schedule It deals with the 22 official languages recognized by the Constitution of
India:
 Assamese
 Bengali
 Bodo
 Dogri (Dongri)
 Gujarati
 Hindi
 Kannada
 Kashmiri
37
MISSION SHOURYA PSI TEST SERIES -2024
 Konkani
 Mathili (Maithili)
 Malayalam
 Manipuri
 Marathi
 Nepali
 Oriya
 Punjabi
 Sanskrit
 Santhali
 Sindhi
 Tamil
 Telugu
 Urdu
Ninth Schedule  It deals with the state acts and regulations of that deal with land reforms
and abolition of the zamindari system. It also deals with the acts and
regulations of the Parliament dealing with other matters.
Note:
 1st Amendment Act 1951 added the Ninth Schedule to protect the laws
included in it from judicial scrutiny on the ground of violation of
fundamental rights.
 However, in 2007, the Supreme Court ruled that the laws included in
this schedule after
 April 24, 1973, are now open to judicial review
Tenth Schedule  It contains provisions relating to disqualification of the members of
Parliament and State Legislatures on the ground of defection.
Note:
 This schedule was added by the 52nd Amendment Act of 1985, also
known as Anti-defection Law
Eleventh Schedule  It contains the provisions that specify the powers, authority and
responsibilities of Panchayats. It has 29 matters.
Note:
 This schedule was added
 by the 73rd Amendment Act of 1992
Twelfth Schedule  It deals with the provisions that specify the powers, authority and
responsibilities of Municipalities. It has 18 matters.
 Note:
 This schedule was added
 by the 74th Amendment Act of 1992

39. Which of the following committees is/ are related to the Centre-State relations in india?
1. Rajamannar Committee
2. Sarkaria commission
3. M.M. Punchhi Commission
Select the correct answer using the code given below:
a) 1 only
b) 1 and 2 only
c) 3 only
d) 1, 2 and 3
Ans: (d) 1, 2 and 3
Committees on Centre-State Relations in India
38
MISSION SHOURYA PSI TEST SERIES -2024
 Rajamannar Committee.
 Anandpur Sahib Resolution.
 West Beneal Memorandum.
 Sarkaria commission.
 MM Punchhi Commission.

40. Arrange the following rivers of Indus river system from North to South:
1. Chenab
2. Jhelum
3. Sutlej
Select the correct answer using the code given below:
a) 1-2-3
b) 1-3-2
c) 2-1-3
d) 2-3-1
Ans: (c) 2-1-3
Indus River
 The Indus is the main river of this river system and originates from the Kailash range glaciers near
Mansarovar lake. The length of the Indus river is around 2880 km, out of which only 710 km lies in the
Indian territories in Jammu & Kashmir state, and the rest lies in Pakistan and Tibet collectively.
 originates from a glacier near Bokhar Chu- Near Manasarovar lake, Tibet. Lies the ranges and foothills
of the Himalayas, the Hindu Kush, and the Karakoram - Range; the rest are the semiarid plains of
Pakistan.
 Tributaries:
o Zaskar (Ladakh)
o Astor (Nanga Parbat)
o Chenab, Sutlej, Soan, Beas, Ravi,Dras, Suru (Indus), Jhelum, Kishanganga, R. Shyok, Gilgit,
o Shigar/Sangar+Kabul+ Kurram Gomal - Tochi +Viboa- Originate in Sulaiman Ranges
o Harike Barrage, at the confluence of the Beas and Sutlej: Channels water into the Indira
Gandhi
 Cuts across the Ladakh range, forming a spectacular gorge near Gilgit in Jammu and Kashmir
 Canyons: in the vicinity of the Nanga Parbat formation
 Tidal Bore Elongated protruding distributaries and low sandy beaches
 Indus flows only through Leh district and gets into Pakistan through the
 Northern Areas- near Chillar in the Dardistan region
 Rivers Chenab, Jhelum, Sutlej, Ravi, Beas+ 2 tributaries from the Afghanistan & Khyber Pakhtunkhwa
 Known as the Sapta Sindhu (Seven Rivers) delta of Pakistan
 Fish: Hilsa
 Blind Indus River Dolphin (Platanista indicus minor) is a sub-species of dolphin found only in the Indus
River.
 sequence of tributaries from north to south is Jhelum, Chenab, Ravi, Beas, Sutlej.

39
MISSION SHOURYA PSI TEST SERIES -2024

41. Which of the following temples is known in the inscriptions as "Dakshina Meru"?
a) Airavatesvara temple in Thanjavur
b) Virupaksha temple in Hampi
c) Brihadeshwara temple in Thanjavur
d) Hoysaleswara temple at Halebidu.
Ans: (c) Brihadeshwara temple in Thanjavur
Brihadisvara Temple
 Brihadeshwara Temple, also known as Raja Rajeswara Temple, is dedicated to Shiva and is located in
Thanjavur (Tamil Nadu).
 Thanjavur lies in the delta of the Cauvery River.
 It is one of the best examples of the Dravidian style of temple architecture built by Chola emperor Raja
Raja Chola I ( 985-1014 AD) between 1003 AD and 1010 AD.
 The Dravidian style of temple architecture flourished roughly from the 8th century to about the 13-14
century AD.
 The temple consists of a pyramidal spire and is adorned with sculptures and paintings inside as well as
outside.
 The temple entered the UNESCO World Heritage Site list in 1987 and is also a part of "Great Living
Chola Temples" along with the Brihadisvara Temple at Gangaikondacholapuram and the Airavatesvara
Temple at Darasuram.

42. He established Poona Sarvajanik Sabha in 1870. He has written some of the books like Rise of
Maratha Power and Essays in India Economics. He was one of the key members of Prarthana
samaj. He regularly voiced views on social and economic reforms at the annual sessions of the
Indian National Social Conference, which he founded in 1887. " The description given above best
suits with reference to:
a) Mahadev Govind Ranade
b) Gopal Krishna Gokhle
c) Atmaram Pandurang
d) Firozshah Mehta
Ans: (a) Mahadev Govind Ranade
Mahadev Govind Ranade
 Mahadev Govind Ranade was born in 1842 at Nashik district Maharashtra. He passed his law degree
and then was appointed as a judge and then became a judge at Bombay HC. He was a distinguished
Indian scholar, social reformer and author.
 He was one of the founding members of the Prarthana Samaj and propounded against prevalent social
evils.
 He would also edit a Bombay Anglo-Marathi daily paper, the Induprakash, founded on his ideology of
40
MISSION SHOURYA PSI TEST SERIES -2024
social and religious reform.
 He educated his wife Ramabai who later became a doctor and also was one of the founders of Seva
Sadan which pioneered women’s rights movements.
 He was also a great educationist and found a number of schools.
 Ranade was a founder of the Social Conference movement, which he supported till his death, directing
his social reform efforts against child marriage, the shaving of widows’ heads, the heavy cost of
marriages and other social functions, and the caste restrictions on traveling abroad, and he strenuously
advocated widow remarriage and female education.
 He was one of the founders of the Widow Marriage Association in 1861.
 He founded the Poona Sarvajanik Sabha which was a socio political body and later was one of the
originators of the Indian National Congress.

43. The tribunal for the purposes of Foreign exchange can be established by:
a) Parliament only
b) Parliament and state legislatures under Article 323-A
c) Parliament and state legislatures under Article 323-B
d) State legislatures under Article 323-A.
Ans: (c) Parliament and state legislatures under Article 323-B
Tribunals
 Tribunal is an administrative body established for the purpose of discharging quasi-judicial duties. The
original Indian Constitution did not contain provisions with respect to tribunals. The 42nd Amendment
Act of 1976 added Tribunals. It performs a number of functions like adjudicating disputes, determining
rights between contesting parties, making an administrative decision, reviewing an existing
administrative decision and so forth.
Various constitutional provisions related to tribunals in India: Provisions for tribunals is provided
through 42nd amendment by creating Part 14-A in the Constitution. It has two articles:
1. Article 323 A: Article 323-A empowers Parliament to create Administrative Tribunals to adjudicate
disputes regarding recruitment and conditions of service of persons appointed to public services at
Union, State and local level or with any other authority within India.
2. Article 323 B: Article 323-B empowers parliament or state legislatures to set up tribunals for other
matters i.e. taxation, foreign exchange, import and export, Industrial and labour, Land reforms,
Ceiling on urban property, Elections to Parliament and state legislatures, Rent and tenancy right.
3. General features: While Article 323 A anticipates establishment of tribunals for public service
matters only, Article 323 B contemplates establishment of tribunals for certain other matters.
 Under Article 323 A can be established only by Parliament, tribunals under Article 323 B can be
established both by Parliament and state legislatures with respect to matters falling within their
legislative competence.
 Under Article 323 A, only one tribunal for the Centre and one for each state or two or more states may
be established.

44. Kiratarjuniyam is a composition of which among the following poets?


a) Kalidas
b) Dandin
c) Bharavi
d) None of the above
(c) Bharvi
 Scholars have interpreted the story depicted on the panel differently. While some believe that it is the
story of the descent of the Ganga from heaven to earth,others believe that the main story is of
Kiratarjuniya or Arjuna‘s penance, a poetic work by Bharvi which is known to have been
popular in the Pallava court.
Facts about Kalidasa

41
MISSION SHOURYA PSI TEST SERIES -2024
 Kalidasa was a Classical Sanskrit writer, widely regarded as the greatest poet and dramatist in the
Sanskrit language of India.
 Much about his life is unknown, only what can be inferred from his poetry and plays.
 His plays and poetry are primarily based on the Vedas, the Ramayana, the Mahabharata and the
Puranas.
 His surviving works consist of three plays, two epic poems and two shorter poems.
 His works cannot be dated with precision, but they were most likely authored within the 4th–5th century
CE.
 Palaeographic evidence: The earliest paleographical evidence of Kalidasa is found in a Sanskrit
inscription dated c. 473 CE, found at Mandsaur's Sun temple.
 His name, along with that of poet Bharavi, is mentioned in a stone inscription dated 634 C.E. found at
Aihole, located in present-day Karnataka.
 Influenced: Kalidasahad great impact on Rabindranath Tagore. Meghadutam's romanticism is found in
Tagore's poems on the monsoons.
 Father of Modern Medicine Sir William Osler always kept on his desk a poem written by Kalidasa.
Literary works
 Kalidasa wrote three plays:
 Malavikagnimitram(Pertaining to Malavika and Agnimitra) tells the story of King Agnimitra, who
falls in love with the picture of an exiled servant girl named Malavika.
 Abhijnanasakuntalam (Of the recollection of Shakuntala) tells the story of King Dushyanta who,
while on a hunting trip, meets Shakuntala, the adopted daughter of a sage, and marries her. It was
among the first Sanskrit works to be translated into English.
 Vikramorvasiyam (Urvashi Won by Valour) tells the story of mortal King Pururavasand celestial
nymph Urvashi who falls in love.
 Kalidasa is the author of two epic poems:
 Raghuvamsa is an epic poem about the kings of Raghu dynasty.
 Kumarasambhava describes the birth and adolescence of the goddess Parvati, and her marriage
with Lord Shiva.
 Kalidasa also wrote two khandakavyas (minor poems):
 Ritusamharadescribes the six seasons by narrating the experiences of two lovers in each of the
seasons.
 Meghaduta(The Cloud Messenger)describes the story of a Yaksha trying to send a message to his
lover through a cloud. This poem is elegiac in nature through which Kalidasa created his own
genre of poetry. Kalidasa set this poem to the mandakranta meter, which is known for its lyrical
sweetness.

45. During the period of Emperor Akbar, Ibadat Khana was a place used for
a) Discussions of religion by scholars of different religions.
b) Prayer hall exclusively for a particular religion.
c) It is hall where Urs (death anniversary of Nizamuddin Auliya) is celebrated.
d) None of the above
(a) Discussions of religion by scholars of different religions.
 While Akbar was at FatehpurSikri during the 1570s he started discussions on religion with the ulama,
Brahmanas, Jesuit priests who were Roman Catholics, and Zoroastrians. These discussions took place in
the ibadatkhana.
Akbar(1556-1605)
 Akbar was only thirteen years old at the time of Humayun’s death. When his father died, Akbar was at
Kalanaur in Punjab and therefore his coronation took place in Kalanaur itself in 1556.
 It was his tutor and Humayun’s favourite and confidant Bairam Khan, who served as the regent of the
Mughal emperor from 1556 to 1560.
42
MISSION SHOURYA PSI TEST SERIES -2024
 As a conqueror Akbar triumphed all over North India. The first four years of Akbar’s
rule saw the expansion of the Mughal empire from Kabul to Jaunpur, including
Gwalior and Ajmer, under his regent Bairam Khan.
 One of the major achievements of his regency period was the defeat of Hemu and the Afghan forces in
the second battle of Panipat in 1556, who were posing a serious threat to the Mughal Empire.
Akbar’s Military Conquests:
 Malwa was conquered in 1562 from Baz Bahadur who was made a mansabdar in Akbar’s court.
 The Gondwana region of central India was annexed after a fierce battle with Rani Durgavati and her son
Vir Narayan in 1564.
 Akbar conquered Gujarat from Muzaffar Shah in 1573.Akbar built new capital Fathepur sikri in
memory of this victory.
 In the Battle of Haldighati, Rana Pratap Singh was severely defeated by the Mughal army led by
Man Singh in 1576. Following the defeat of Mewar, most of the leading Rajput rulers had accepted
Akbar’s suzerainty.
 Defeated Daud Khan, the Afghan ruler of Bihar and Bengal, both the provinces were annexed to the
Mughal empire in 1576.
 Akbar’s forces had occupied Khandesh region in 1591.
 Chand Bibi defended Ahmednagar against the Mughal forces.

46. The amara-nayakas of the Vijaynagara Kingdoms were:


a) Bureaucrats
b) Temple priest
c) Military commanders
d) None of the above
C. Military commanders
 The amara-nayakas were military commanders who were given territories to govern by the Raya. They
collected taxes and other dues from peasants, craftspersons and traders in the area. They retained part of
the revenue for personal use and for maintaininga stipulated contingent of horses and elephants.
Administration in Vijaynagara Kingdoms
 They followed traditional monarchy.
 The king was the ultimate authority in the kingdom. He was also the supreme commander of the army.
 The king was assisted by a council of ministers in his day to day administration.
 One of the important characteristics of the Vijayanagar administration was the amaranayaka This is
similar to the iqta system of Delhi Sultanate.
 In this system, the commander of the Vijayanagar army was called the nayaka. Each nayaka was given
an area for administration. The nayaka was responsible for expanding agricultural activities in his area
 He collected taxes in his area and with this income maintained his army, horses, elephants and
weapons of warfare that he had to supply to the raya or the Vijayanagar ruler
 The amara-nayakas sent tribute to the king annually and personally appeared in the royal court with
gifts to express their loyalty.
Revenue:
 Land revenue was the major source of income to the state.
 Genrally it was it was at the rate of one sixth of the total produce.

47. Dipavamsa (the chronicle of Island) and Mahavamsa (the great chronicle) areassociated with:
43
MISSION SHOURYA PSI TEST SERIES -2024
a) Jainism
b) Sangam literature
c) Buddhism
d) Upanishads
(c) Buddhism
 As Buddhism travelled to new regions such as Sri Lanka, other texts such as the Dipavamsa
(literally, the chronicle of the island) and Mahavamsa (the great chronicle) were written, containing
regional histories of Buddhism.Many of these works contained biographies of the Buddha. Some of the
oldest texts are in Pali, while later compositions are in Sanskrit.
Buddhism
 Buddhism was founded by Gautama in 566 B.C. He was the son of Suddhodhan and Mayadevi. His
father Suddhodhan was the eminent king of Sakya republic.
 Gautama was married to Yasodhara at an early age. Gautama was shocked at the sight of an old man, a
diseased person, a dead body.
 Thereafter, he was attracted by the saintly appearance of an ascetic. One night he renounced the worldly
life and left his home, wife, and son.
 After leaving home, Gautama studied for some time in the philosophical schools of two renowned
teachers. Thereafter, six years of profound meditation led to the discovery of truths. Gautama became
the ‘Buddha's.e. the enlightened one.
The main principle of Buddha’s teachings is represented by the “Four Noble Truths (Arya-Satya's)” as
:
 Dukkha (the world is full of sorrow)
 Dukkha Samuddaya (that causes the sorrow)
 Dukkha Nirodha (sorrow can be stopped)
 Dukkha Nirodhagamini-pratipada (the path leading to the cessation of sorrow)
Teachings of Buddha
 Desire is the root of all human sadness and the assured way of ending unhappiness was the elimination
of desire.
 Death is certain and there is no escape from it which leads to rebirth and caused further suffering. One
could get out of this chain of suffering by attaining Nirvana (salvation).
 To achieve the final salvation (Nirvana) Buddha suggests ‘Ashtangika marga’ (the eight-fold path).
These eightfold paths are :
 Right speech,
 Right action,
 Right means of livelihood,
 Right exertion,
 Right mindfulness,
 Right meditation,
 Right resolution, and
 Right view.
 The ultimate aim of life is to attain Nirvana, which means freedom, from further birth and death.
Nirvana is the eternal state of peace and bliss or liberation from the cycle of birth and death.
Buddha had summarized the whole process in three words viz:
 Seela (Right conduct),
44
MISSION SHOURYA PSI TEST SERIES -2024
 Samadhi (Right concentration), and
 Prajna (Right knowledge).
 According to Buddha, Seela and Samadhi lead to Prajna, which is the direct cause of nirvana.
 Buddha advocated “The Middle Path” in which extremes are avoided.
 Buddha visited the Deer Park (modern Sarnath), Kasi after his enlightenment, and gave his first
Sermon (lecture).
 His first sermon was known as “Set in Motion, the Wheel of Law”.

The followers of the Buddha fell into two categories :


 Upasakas i.e. the lay followers who lived with family; and
 Bhikshus i.e. monks who renounced the world and led the life of an ascetic.
Buddhist councils

48. Which of the following would be the effect of increase in Bank Rate?
a) It will increase the cost of borrowing by commercial banks.
b) It will reduce the supply of money.
c) It will show tightening of RBI monetary policy.
d) All of the above.
(d) All of the above.
Bank Rate refers to the official interest rate at which Central Bank (RBI) will provide loans to the banking
system which includes commercial / cooperative banks,development banks etc.
 Central Bank buys or rediscounts bills of exchange or other commercial papers at Bank Rate. It also
signals the medium-term stance of monetary policy.
 Bank rate is used as a signal by the RBI to the commercial banks on RBI’s thinking of what the interest
rates should be. When bank rate is raised, it is expected that all interest rates will move together in the
same direction.
Difference between Bank Rate and Repo Rate:
 Bank Rate and Repo Rate seem to be similar terms because in both of them RBI lends to the banks.
However, Repo Rate is a short-term measure and it refers to short-term loans and used for controlling
the amount of money in the market. On the other hand, Bank Rate is a long-term measure and is
governed by the long-term monetary policies of the RBI.
Policy Rates:
1. Bank rate – The interest rate at which RBI lends long term funds to banks is referred to as the bank
45
MISSION SHOURYA PSI TEST SERIES -2024
rate. However, presently RBI does not entirely control money supply via the bank rate. It uses
Liquidity Adjustment Facility (LAF) – repo rateas one of the significant tools to establish control over
money supply.Bank rate is used to prescribe penalty to the bank if it does not maintain the prescribed
SLR or CRR
2. Liquidity Adjustment Facility (LAF) – RBI uses LAF as an instrument to adjust liquidity and
money supply. The following types of LAF are:
1. Repo rate: Repo rate is the rate at which banks borrow from RBI on a short-term basis against a
repurchase agreement. Under this policy, banks are required to provide government securities as
collateral and later buy them back after a pre-defined time.
2. Reverse Repo rate: It is the reverse of repo rate, i.e., this is the rate RBI pays to banks in order
to keep additional funds in RBI. It is linked to repo rate in the following way:
Reverse Repo Rate = Repo Rate – 1
3. Marginal Standing Facility (MSF) Rate: MSF Rate is the penal rate at which the Central Bank
lends money to banks, over the rate available under the rep policy. Banks availing MSF Rate can use
a maximum of 1% of SLR securities.

49. Which of the following sector of Indian economy has highest labour force?
a) Primary sector.
b) Secondary sector.
c) Tertiary sector.
d) Quaternary Sector.
Ans: a) Primary sector.
 The primary sector employs the largest portion of the Indian population. In the Primary sector of the
economy, activities are undertaken by directly using natural resources.
Primary Sector
 In Primary sector of economy, activities are undertaken by directly using natural resources.
Agriculture, Mining, Fishing, Forestry, Dairy etc. are some examples of this sector.
 It is called so because it forms the base for all other products. Since most of the natural products we
get are from agriculture, dairy, forestry, fishing, it is also called Agriculture and allied sector.
 People engaged in primary activities are called red-collar workers due to the outdoor nature of their
work.
Secondary Sector
 It includes the industries where finished products are made from natural materials produced in the
primary sector. Industrial production, cotton fabric, sugar cane production etc. activities comes under
this sector.
 Hence its the part of a country's economy that manufactures goods, rather than producing raw materials
 Since this sector is associated with different kinds of industries, it is also called industrial sector.
 People engaged in secondary activities are called blue collar workers.
 Examples of manufacturing sector:
o Small workshops producing pots, artisan production.
o Mills producing textiles,
o Factories producing steel, chemicals, plastic, car.
o Food production such as brewing plants, and food processing.
o Oil refinery.

Tertiary Sector/Service Sector


 This sector’s activities help in the development of the primary and secondary sectors. By itself,
economic activities in tertiary sector do not produce a goods but they are an aid or a support for the
production.
 Goods transported by trucks or trains, banking, insurance, finance etc. come under the sector. It
46
MISSION SHOURYA PSI TEST SERIES -2024
provides the value addition to a product same as secondary sector.
 This sector jobs are called white collar jobs.
Quaternary Activities
 These are specialized tertiary activities in the ‘Knowledge Sector’ which demands a separate
classification.
 The quaternary sector is the intellectual aspect of the economy. It is the process which enables
entrepreneurs to innovate and improve the quality of services offered in the economy.
 Personnel working in office buildings, elementary schools and university classrooms, hospitals and
doctors’ offices, theatres, accounting and brokerage firms all belong to this category of services.
 Like other tertiary functions, quaternary activities can also be outsourced.
Quinary Activities
 The quinary sector is the part of the economy where the top-level decisions are made. This includes
the government which passes legislation. It also comprises the top decision-makers in industry,
commerce and also the education sector.
 These are services that focus on the creation, re-arrangement and interpretation of new and
existing ideas; data interpretation and the use and evaluation of new technologies.
 Profession under this category often referred as 'gold collar' professions, they represent another
subdivision of the tertiary sector representing special and highly paid skills of senior business
executives, government officials, research scientists, financial and legal consultants, etc.

50. Which of the following are called Brettonwoods Twins?


a) IMF and International Bank for Reconstruction and Development (IBRD)
b) IMF & World Trade Organisation
c) IBRD & World Trade Organisation
d) None of the above
Ans: (a) IMF & International Bank for Reconstruction and Development (IBRD)
Brettonwoods Twins
 IMF & IBRD was founded altogether in 1946 in the US. This is why they are called Brettonwoods
Twins.The Bretton Woods twins refer to the two multilateral organizations created at the Bretton Woods
Conference in 1944. They are: The World Bank (IBRD), The International Monetary Fund.
World Bank
 The International Bank for Reconstruction and Development (IBRD) and the International
Development Association (IDA), two of the World Bank Group's five international entities, are known
as the World Bank.
 The World Bank is an international financial institution that lends and gives money to the
governments in low- and middle-income nations to fund capital projects. It is headquartered
in Washington D.C, United States.
 It was formed with the International Monetary Fund at the 1944 Bretton Woods Conference.
 The World Bank Group is a global partnership of 189 countries and five constituent organisations
committed to alleviating poverty and promoting prosperity.
 The World Bank Group's five development institutions are:
1. International Bank for Reconstruction and Development (IBRD)
2. International Development Association (IDA)
3. International Finance Corporation (IFC)
4. Multilateral Guarantee Agency (MIGA)
5. International Centre for the Settlement of Investment Disputes (ICSID)
International Monetary Fund (IMF)
The International Monetary Fund (IMF) is an organization of 190-countries, dedicated to global monetary

47
MISSION SHOURYA PSI TEST SERIES -2024
cooperation, financial stability, international trade facilitation, high employment and long-term economic
growth, and poverty reduction
 The Bretton Woods Conference in 1944 kicked off the foundation of the IMF.
 The International Monetary Fund (IMF) was founded on December 27, 1945, and it now has 189
member countries.
 The International Monetary Fund (IMF), based in Washington, D.C., is dedicated to developing global
monetary cooperation, ensuring financial stability, and facilitating and promoting international
commerce, employment, and economic progress.
 The International Monetary Fund (IMF) is a United Nations specialised agency.
Objectives of IMF
 Encourage international monetary cooperation.
 Assurance of Financial stability.
 Make international trade easier.
 Encourage high employment and long-term economic prosperity.
 Reduce poverty all around the planet.
Special Drawing Rights (SDRs)
 The SDR is neither a currency nor a claim on the International Monetary Fund. Rather, it is a
prospective claim on IMF members' freely useable currencies. These currencies can be exchanged for
SDRs.
 The IMF and a few other international organizations use the SDR as their accounting unit.
 The SDR's currency value is calculated by adding the values of an SDR basket of currencies in US
dollars, based on market exchange rates.
 The US dollar, Euro, Japanese yen, pound sterling, and Chinese renminbi are all included in the SDR
basket of currencies (included in 2016).
 The SDR currency value is updated every five years (except during IMF vacations or when the IMF is
closed for business), and the valuation basket is reviewed and altered every five years.
 A country's quota (the amount it contributes to the IMF) is measured in SDRs.
 The voting power of members is proportional to their quotas.
 The IMF distributes SDRs to its members in proportion to their existing quotas in the organization.
 Aside from gold reserves, foreign currency assets, and the IMF Reserve Tranche, India's foreign
exchange reserves include SDR.
India & IMF
 India is a founding member of the International Monetary Fund.
 India's Union Finance Minister serves on the IMF's Board of Governors as an Ex Officio Governor.
 Each member country also has a governor who serves as an alternate.
 The Governor of the Reserve Bank of India serves as India's alternate governor.
 In addition, India has an Executive Director who represents the country at the IMF.
 India's shareholding in the IMF is 2.76%, with a quota of SDR 13,114.4 million.
 In the linked article, you may learn more about the Special Drawing Rights (SDRs), which were created
by the International Monetary Fund (IMF) in 1969.
 As a result, India is the organization's eighth-largest quota holder.
 In the year 2000, India finished repaying all of the IMF loans it had taken out.
 India is now a contributor to the International Monetary Fund (IMF).
 Emerging economies have gained increasing clout in the International Monetary Fund's governance
structure (IMF).
 The reforms were agreed upon by the IMF's then 188 members in 2010, following the global financial
crisis.
 More than 6% of quota shares will be transferred from the United States and Europe to emerging and
developing countries.

48
MISSION SHOURYA PSI TEST SERIES -2024
51. Which of the following Biosphere Reserves is incorrectly matched with the state in which it is
located?
a) Dehang- Dibang - Arunachal Pradesh
b) Pachmarhi – Odisha
c) Panna - Madhya Prades
d) Nokrek – Meghalaya
Ans: (b) Pachmarhi - Odisha
Pachmarhi is located in Madhya Pradesh
Biosphere Reserves
 Cold Desert, Himachal Pradesh
 Nanda Devi, Uttrakhand
 Khangchendzonga, Sikkim
 Dehang-Debang, Arunachal Pradesh
 Manas, Assam
 Dibru-Saikhowa, Assam
 Nokrek, Meghalaya
 Panna, Madhya Pradesh
 Pachmarhi, Madhya Pradesh
 Achanakmar-Amarkantak, Madhya Pradesh-Chhattisgarh
 Kachchh, Gujarat (Largest Area)
 Similipal, Odisha
 Sundarban, West Bengal
 Seshachalam, Andhra Pradesh
 Agasthyamala, Karnataka-Tamil Nadu-Kerala
 Nilgiri, Tamil Nadu-Kerala (First to be Included)
 Gulf of Mannar, Tamil Nadu
 Great Nicobar, Andaman & Nicobar Island

52. Who among the following coined the slogans of “Quit India” and “Simon Go Back”?
a) Yusuf Meherally
b) Rahmat Ali
c) Surayya Tyabji
d) Pingali Venkayya
Ans: (a) Yusuf Meherally
Quit India Movement On August 8, 78 years ago, Mahatma Gandhi gave the call for British colonizers to
“Quit India” and for the Indians to “do or die” to make this happen. Soon after, Gandhi and almost the entire
top Congress leadership was arrested, and thus began a truly people-led movement in our freedom struggle,
eventually quelled violently by the British, but leaving behind a clear message– the British would have to
leave India, and no other solution would be acceptable to its masses.
 While factors leading to such a movement had been building up, matters came to a head with the failure
of the Cripps Mission.The objective of Cripps Mission was to secure India’s whole-hearted support in
the war, in return for self-governance.
 However, despite the promise of “the earliest possible realization of self-government in India”, the offer
Cripps made was of dominion status, and not freedom. Also, there was a provision of the partition of
India, which was not acceptable to the Congress. On August 8, Bapu addressed the people from
Mumbai’s Gowalia Tank maidan. “Here is a mantra, a short one, that I give you. Imprint it on your
hearts, so that in every breath you give expression to it. The mantra is: ‘Do or Die’. We shall either free
India or die trying; we shall not live to see the perpetuation of our slavery,” Gandhi said.
 While Gandhi gave the clarion call of Quit India, the slogan was coined by Yusuf Meherally, a socialist
and trade unionist who also served as Mayor of Mumbai. A few years ago, in 1928, it was Meherally
who had coined the slogan

49
MISSION SHOURYA PSI TEST SERIES -2024
53. Consider the following pairs:
1. Producers – Autotrophs
2. Primary Consumers - Herbivores
3. Decomposers - Saprotrophs
Which of the above pairs are correctly matched?
a) 1 and 2 only
b) 2 and 3 only
c) 1 and 3 only
d) 1, 2 and 3
Ans: (d) 1, 2 and 3
Biotic Components
 Biotic components include living organisms comprising plants, animals and microbes and are classified
according to their functional attributes into producers and consumers.
 Primary producers – Autotrophs (self-nourishing) Primary producers are basically green plants (and
certain bacteria and algae).They synthesise carbohydrate from simple inorganic raw materials like
carbon dioxide and water in the presence of sunlight by the process of photosynthesis for themselves,
and supply indirectly to other non-producers. In terrestrial ecosystem, producers are basically
herbaceous and woody plants, while in aquatic ecosystem producers are various species of microscopic
algae.
 Consumers – Heterotrophs :Consumers are incapable of producing their own food (photosynthesis).
They depend on organic food derived from plants, animals or both.Herbivores are primary consumers
which feed mainly on plants e.g. cow, rabbit.
 Secondary consumers feed on primary consumers e.g. wolves., known as carnivores. Carnivores
which feed on secondary consumers are called tertiary consumers e.g. lions which can eat wolves.
Omnivores are organisms which consume both plants and animals e.g. man, monkey.
 Decomposers- Saprotrophs They are bacteria and fungi which obtain energy and nutrients by
decomposing dead organic substances (detritus) of plant and animal origin. The products of
decomposition such as inorganic nutrients which are released in the ecosystem are reused by producers
and thus recycled. Earthworm and certain soil organisms (such as nematodes, and arthropods) are
detritus feeders and help in the decomposition of organic matter and are called detrivores.

54. Which of the following pairs are correctly matched?


Regions Separated by
1. Indira Point and A. 6° channel
Indonesia
2. Maldives and Minicoy B. 9° channel
3. Lakshadweep and C. 8° channel
Minicoy
4. Andaman and Nicobar D. 10° channel
Select the correct answer using the code below:
a) 1, 2 and 3 only
b) 1, 3 and 4 only
c) 1 and 4 only
d) 1, 2, 3 and 4
Ans: (c) 1 and 4 only
Regions Separated by
 Indira Point and Indonesia: 6° channel
 Maldives and Minicoy: 8° channel
 Lakshadweep and Minicoy: 9° channel
 Andaman and Nicobar channel: 10° channel

50
MISSION SHOURYA PSI TEST SERIES -2024
55. Which of the following passes and the regions are matched correctly?
1. Zoji La - Great Himalayas
2. Photu La – Zaskar
3. Banihal -Pir Panjal
Select the correct answer using the code given below:
a) 1 and 2 only
b) 2 and 3 only
c) 1 and 3 only
d) 1, 2 and 3
Ans: (d) 1,2 and 3
 Kashmir or Northwestern Himalayas It comprise a series of ranges such as the Karakoram, Ladakh,
Zaskar and Pir Panjal. The northeastern part of the KashmirHimalayas is a cold desert, which lies
between the Greater Himalayas and the Karakoram ranges.
 Between the Great Himalayas and the Pir Panjal range, lies the world famous valley of Kashmir and the
famous Dal Lake. Important glaciers of South Asia such as the Baltoro and Siachen are also found in
this region.
 The Kashmir Himalayas are also famous for Karewa formations, which are useful for the cultivation of
Zafran, a local variety of saffron.
 Some of the important passes of the region are Zoji La on the Great Himalayas, Banihal on the
Pir Panjal,Photu La on the Zaskar and Khardung la on ladakh range

56. What is the chemical name of quick lime


a) Calcium oxide
b) Calcium hydroxide
c) Calcium carbonate
d) Calcium bicarbonate
Ans: (a) Calcium oxide
Calcium oxide (CaO) is commonly called quick lime.
List of the Important Salt and their Uses
Name of Salts Uses
Sodium Chloride or Common Salt  Used in cooking food as well as for cooking gas.
(NaCl)  Used as a preservative in pickles and in curing meat and
fish.
 Used in the manufacture of soap.
 Used to melt the ice in winter in cold countries.
 Used for making chemical likes washing Soda, baking soda
etc.
Sodium Hydroxide (NaOH)  Used for making soap and detergent
 Used for making artificial textile fibre (rayon)
 Used in the manufacturing of paper
 Used for purification of bauxite ore
 Used in de-greasing metals, oil refining and making dyes
and bleaches
Sodium Carbonate or Washing Soda  Used as cleansing agent
(Na2CO3.10H2O)  Used for removing permanent hardness of water
 Used in the manufacturing of glass, soap and paper
Baking Soda or Sodium Bi-  Used as neutralising agent (antacid)
carbonate (NaHCO3)  Used for making baking powder
 Used in fire extinguisher
Bleaching Powder or Calcium  Used as bleaching agent in cotton industry and paper
51
MISSION SHOURYA PSI TEST SERIES -2024
Hypochlorite industry for bleaching cotton and linen as well as wood
pulp respectively
 Used for disinfecting drinking water
 Used for making chloroform (CHCl3)
 Used for making the wool unshrinkable
Plaster of Paris or Hemihydrate  Used in hospital for setting fractured bone
Calcium sulphate, CaSO4 1/2 H2O  Used for making toys, decoration material cheap ornament,
chalk etc
 Used for fire-proofing material
 Used for making surface smooth.
57. A change of state directly from solid to gas without changing into liquid state is called:
a) Fusion
b) Sublimation
c) Diffusion
d) Emulsion
Ans: (b) Sublimation
Sublimation is the transition of a substance directly from the solid to the gas phase without passing through
the intermediate liquid phase.
 Evaporation is a process by which water is transformed from liquid to gaseous state. Heat is the main
cause for evaporation.
 The temperature at which the water starts evaporating is referred to as the latent heat of vapourisation.
The transformation of water vapour into water is called condensation.
 Condensation is caused by the loss of heat. When moist air is cooled, it may reach a level where its
capacity to hold water vapour ceases.
 Then, the excess water vapour condenses into liquid form. If it directly condenses into solid form, it is
known as sublimation.

58. Which of the following is used as a lubricant?


a) Graphite
b) Silica
c) Iron Oxide
d) Diamond
Ans:A Graphite
Graphite is used as a lubricant.
 The slippery substance is used extensively as a lubricant.
 Graphite has a loosely bounded C-C bond so, it is slippery and used as a lubricant.
 Silica, iron oxide and Diamond are hard solids and do not have slippery nature so, this is not used as
lubricants.
Graphite
 Graphite is a naturally occurring form of crystalline carbon.
 Graphite is also known as plumbago or black lead or mineral carbon is a stable form of naturally
occurring carbon.
 The carbon content in Graphite is never less than 95%.
 Graphite may be considered the highest grade of coal, just above anthracite.
 It is not normally used as fuel because it is difficult to ignite.
 It is found in metamorphic and igneous rocks.
 Graphite is extremely soft, cleaves with very light pressure.
 It is extremely resistant to heat and is highly unreactive.
 Most of the graphite is formed at convergent plate boundaries where organic-rich shales and

52
MISSION SHOURYA PSI TEST SERIES -2024
limestones were subjected to metamorphism due to heat and pressure.
 Metamorphism produces marble, schist, and gneiss that contain tiny crystals and flakes of graphite.
 Some graphite forms from the metamorphism of coal seams. This graphite is known as “amorphous
graphite”.
 Graphite is a non-metal and it is the only non-metal that can conduct electricity.
Applications of Graphite
 Natural graphite is mostly consumed for refractories, batteries, steelmaking, expanded graphite,
lubricants, etc.
 A refractory material is one that retains its strength at high temperatures.
 Natural and synthetic graphite are used to construct the anode of all major battery technologies.
 The lithium-ion battery utilizes roughly twice the amount of graphite than lithium carbonate.
 Natural graphite in this end-use mostly goes into carbon raising in molten steel [to make steel stronger]
 Natural amorphous graphite are used in brake linings for heavier vehicles and became important with
the need to substitute for asbestos.
 Graphite lubricants are specialty items for use at very high or very low temperatures.
 Modern pencil lead is most commonly a mix of powdered graphite and clay.

59. Which of the following is the „master gland?


a) Thymus gland
b) Pancreas gland
c) Pineal gland
d) Pituitary gland
Ans: (D) Pituitary gland
 The pituitary is a small endocrine gland located below the hypothalamus in the brain. It is called as
master gland because its secretion regulates and controls the secretion of other endocrine glands such as
thyroid gland, adrenal gland, pancreas etc.

53
MISSION SHOURYA PSI TEST SERIES -2024
60. Ranikhet disease is associated with
a) Fish
b) Honeybees
c) Hens
d) Pigs
Ans: (C) Hens
 Ranikhet which is also known as New Castle disease or Doyle's disease is an acute, infectious & highly
contagious disease of hens, characterized by respiratory Distress, nervous symptoms & high mortality.
It is caused by Newcastle Disease Virus (NDV).

61. Which of the following parts of the constitution does not contain any provision indicating that
India is secular state?
a) Preamble
b) Fundamental Rights
c) Directive Principles
d) Fundamental Duties
Ans: (d) Fundamental Duties
 The Preamble to the Indian Constitution is based on the ‘Objectives Resolution’, drafted and moved by
Pandit Nehru, and adopted by the Constituent Assembly. It has been amended by the 42nd
Constitutional Amendment Act (1976), which added three new words —socialist, secular and integrity.
By this amendment what was implicit was made explicit. The Preamble itself spoke of liberty of
thought, expression, belief, faith and worship. While granting this liberty the Preamble promised
equality of status and opportunity. So, option (a) is not correct.
 The term ‘secular’ too was added by the 42nd Constitutional Amendment Act of 1976. However, as the
Supreme Court said in 1974, although the words ‘secular state’ were not expressedly mentioned in the
Constitution, there can be no doubt that Constitution-makers wanted to establish such a state and
accordingly Articles 25 to 28 (guaranteeing the fundamental right to freedom of religion) have been
included in the constitution. So, option (b) is not correct.
 The phrase ‘Directive Principles of State Policy’ denotes the ideals that the State should keep in mind
while formulating policies and enacting laws. These are the constitutional instructions or
recommendations to the State in legislative, executive and administrative matters. In the DPSP, the
articles of 38, 39, 39A, 41 & 46 not only attempts to promote equal opportunity for growth and
sustenance for all, but these principles, coupled with the most basic objective of the state, the doctrine of
‘Parens Patriae’, promotes secularism in all of its form. So, option (c) is not correct.
 Sardar Swaran Singh Committee recommended the inclusion of a separate chapter on fundamental
duties in the Constitution. It stressed that the citizens should become conscious that in addition to the
enjoyment of rights, they also have certain duties to perform as well. The Congress Government at
Centre accepted these recommendations and enacted the 42nd Constitutional Amendment Act in 1976.
This amendment added a new part, namely, Part IVA to the Constitution. This new part consists of only
one Article, that is, Article 51A which for the first time specified a code of ten fundamental duties of the
citizens. But it does not contain any provisions indicating that India is a secular state. So, option (d) is
correct.
FUNDAMENTAL DUTIES
According to Article 51 A, it shall be the duty of every citizen of India:
 To abide by the Constitution and respect its ideals and institutions, the National Flag and the National
Anthem;
 To cherish and follow the noble ideals that inspired the national struggle for freedom;
 To uphold and protect the sovereignty, unity and integrity of India;
 To defend the country and render national service when called upon to doso;
 To promote harmony and the spirit of common brotherhood amongst all the people of India
transcending religious, linguistic and regional or sectional diversities and to renounce practices
derogatory to the dignity of women;
54
MISSION SHOURYA PSI TEST SERIES -2024
 To value and preserve the rich heritage of the country’s composite culture;
 To protect and improve the natural environment including forests, lakes, rivers and wildlife and to have
compassion for living creatures;
 To develop scientific temper, humanism and the spirit of inquiry and reform;
 To safeguard public property and to abjure violence;
 To strive towards excellence in all spheres of individual and collective activity so that the nation
constantly rises to higher levels of endeavour and achievement; and
 To provide opportunities for education to his child or ward between the age of six and fourteen years.
This duty was added by the 86thConstitutional Amendment Act, 2002.

62. Which one of the following statements is correct with reference to the right to education under the
Article 21A of the Indian Constitution?
a) This provision was added by the 84th Constitutional Amendment Act of 2002
b) It declares that the State shall provide free and compulsory education to every child until he
completes the age of 14 years.
c) This provision makes only elementary education a Fundamental Right
d) It mandates 25% reservation for disadvantaged sections of the society
Ans: (c) This provision makes only elementary education a Fundamental Right
 The 42nd Amendment Act of 1976 froze the allocation of seats in the Lok Sabha to the states and the
division of each state into territorial constituencies till the year 2000 at the 1971 level. This ban on
readjustment was extended for another 25 years (ie, upto year 2026) by the 84 th Amendment Act of
2001, with the same objective of encouraging population limiting measures.So, option (a) is not
correct.
 Article 45 in the Constitution of India was set up as an act: The state shall endeavour to provide,within
a period of ten years from commencement of this Constitution, for free and compulsory education for all
children until they complete the age of fourteen years. So, option (b) is not correct. (NOT ARTICLE
21A)
 The primary education in India is divided into two parts, namely Lower Primary (Class I-IV) and Upper
Primary (Middle school, Class V-VIII). The Indian government lays emphasis on primary education
(Class I-VIII) also referred to as elementary education, to children aged 6 to 14 years old.Article 21 A
declares that the State shall provide free and compulsory education to all children of the age of six to
fourteen years in such a manner as the State may determine. Thus, this provision makes only elementary
education a Fundamental Right and not higher or professional education. So, option (c) is correct.
 The Right of Children to Free and Compulsory Education (RTE) Act, 2009, which came into effect
from April 1, 2010 mandates that all private aided, special category schools and private unaided schools
to admit in class I (or below) to the extent of at least 25% of the strength of that class,children belonging
to weaker sections and disadvantaged groups and provide free and compulsory education till its
completion. So, option (d) is not correct.
Right to Education
 Article 21 A declares that the State shall provide free and compulsory education to all children of the
age of six to fourteen years in such a manneras the State may determine.
 Thus, this provision makes only elementary education a Fundamental Right and not higher or
professional education.
 This provision was added by the 86th Constitutional Amendment Act of 2002. This amendment is a
major milestone in the country’s aim to achieve ‘Education for All’. The government described this step
as ‘the dawn of thesecond revolution in the chapter of citizens’ rights’.
 Even before this amendment, the Constitution contained a provision for free and compulsory education
for children under Article 45 in Part IV.
 However, being a directive principle, it was not enforceable by the courts.Now, there is scope for
judicial intervention in this regard.
 This amendment changed the subject matter of Article 45 in directive principles. It now reads— ‘The
state shall endeavour to provide early childhood care and education for all children until they complete
the age of six years.’ It also added a new fundamental duty under Article 51A that reads —‘It shall be
55
MISSION SHOURYA PSI TEST SERIES -2024
the duty of every citizen of India to provide opportunities for education to his child or ward between the
age of six and fourteen years’.
 In 1993 itself, the Supreme Court recognised a Fundamental Right to primary education in the right to
life under Article 21. It held that every child or citizen of this country has a right to free education until
he completes the age of 14 years. There after, his right to education is subject to the limits of economic
capacity and development of the state. In this judgement, the Court overruled its earlier judgement
(1992) which declared that there was a fundamental right to education up to any level including
professional education like medicine and engineering.
 In pursuance of Article 21A, the Parliament enacted the Right of Children to Free and Compulsory
Education (RTE) Act, 2009. This Act seeks to provide that every child has a right to be provided full
time elementary education of satisfactory and equitable quality in a formal school which satisfies certain
essential norms and standards. This legislation is anchored in the belief that the values of equality,
social justice and democracy and the creation of a just and humane society can be achieved only through
provisionof inclusive elementary education to all.

63. Which among the following acts relaxed the central control over the provinces by demarcating and
separating the central and provincial subjects?
a) Indian Councils Act of 1892
b) Government of India Act 1909
c) Government of India Act 1919
d) Government of India Act 1935
(c) Government of India Act 1919
 Indian Councils Act of 1892made a limited and indirect provision for the use of election in filling up
some of the non-official seats both in the Central and provincial legislative councils. So, option (a) is
not correct.
 Government of India Act of 1909considerably increased the size of the legislative councils, both
Centraland provincial. The number of members in the Central Legislative Council was raised from 16 to
60. The number of members in the provincial legislative councils was not uniform. It retained official
majority in the Central Legislative Council but allowed the provincial legislative councils to have non-
official majority. So, option (b) is not correct.
 Government of India Act of 1919 relaxed the central control over the provinces by demarcating and
separating the central and provincial subjects. The central and provincial legislatures were authorised to
make laws on their respective list of subjects. However, the structure of government continued to be
centralised and unitary. So, option (c) is correct.
 Government of India Act 1935 abolished dyarchy in the provinces and introduced ‘provincial
autonomy’ in its place. The provinces were allowed to act as autonomous units of administration in their
defined spheres. Moreover, the Act introduced responsible governments in provinces, that is, the
governor was required to act with the advice of ministers responsible to the provincial legislature. So,
option (d) is not correct.
Government of India Act of 1919
 On August 20, 1917, the British Government declared, for the first time, thatits objective was the
gradual introduction of responsible government inIndia.
 The Government of India Act of 1919 was thus enacted, which came intoforce in 1921. This Act is also
known as Montagu-Chelmsford Reforms(Montagu was the Secretary of State for India and Lord
Chelmsford was theViceroy of India).
Features of the Act
 It relaxed the central control over the provinces by demarcating andseparating the central and provincial
subjects. The central and provincial legislatures were authorised to make laws on their respective list of
subjects. However, the structure of government continued to be centralised and unitary.
 It further divided the provincial subjects into two parts—transferred and reserved. The transferred
subjects were to be administered by the governor with the aid of ministers responsible to the legislative
56
MISSION SHOURYA PSI TEST SERIES -2024
Council.The reserved subjects, on the other hand, were to be administered by the governor and his
executive council without being responsible to the legislative Council. This dual scheme of governance
was known as‘dyarchy’—a term derived from the Greek word di-arche which means double rule.
However, this experiment was largely unsuccessful.
 It introduced, for the first time, bicameralism and direct elections in the country. Thus, the Indian
Legislative Council was replaced by abicameral legislature consisting of an Upper House (Council of
State) anda Lower House (Legislative Assembly). The majority of members of boththe Houses were
chosen by direct election.
 It required that the three of the six members of the Viceroy’s executiveCouncil (other than the
commander-in-chief) were to be Indian.
 It extended the principle of communal representation by providing separate electorates for Sikhs, Indian
Christians, Anglo-Indians and Europeans.
 It granted franchise to a limited number of people on the basis ofproperty, tax or education.
 It created a new office of the High Commissioner for India in London andtransferred to him some of the
functions hitherto performed by the Secretary of State for India.
 It provided for the establishment of a public service commission. Hence, a Central Public Service
Commission was set up in 1926 for recruiting civil servants.
 It separated, for the first time, provincial budgets from the Central budgetand authorised the provincial
legislatures to enact their budgets.
 It provided for the appointment of a statutory commission to inquire intoand report on its working after
ten years of its coming into force.

64. Which one of the following writs shall be issued when the court finds that a particular office holder
is not doing legal duty?
a) Certiorari
b) Quo Warranto
c) Mandamus
d) Prohibition
(c) Mandamus
 Certiorari, in the literal sense, it means ‘to be certified’ or ‘to be informed’. It is issued by a higher
court to a lower court or tribunal either to transfer a case pending with the latter to itself or to squash the
order of the latter in a case. It is issued on the grounds of excess of jurisdiction or lack of jurisdiction or
error of law.So, option (a) is not correct.
 Quo-Warranto, in the literal sense, it means ‘by what authority or warrant’. It is issued by the court to
enquire into the legality of claim of a person to a public office. Hence, it prevents illegal usurpation of
public office by a person.So, option (b) is not correct.
 Mandamus literally means ‘we command’. It is a command issued by the court to a public official
asking him to perform his official duties that he has failed or refused to perform. It can also be issued
against any public body, a corporation, an inferior court, a tribunal or government for the same
purpose.So, option (c) is correct.
 Prohibition literally, it means ‘to forbid’. It is issued by a higher court to a lower court or tribunal to
prevent the latter from exceeding its jurisdiction or usurping a jurisdiction that it does not possess. Thus,
unlike mandamus that directs activity, the prohibition directs inactivity.So, option (d) is not correct.
 Habeas Corpus is a Latin term which literally means ‘to have the body of’. It is an order issued by the
court to a person who has detained another person, to produce the body of the latter before it. The court
then examines the cause and legality of detention. It would set the detained person free, if the detention
is found to be illegal. Thus, this writ is a bulwark of individual liberty against arbitrary detention. The
writ of habeas corpus can be issued against both public authorities as well as private individuals. The
writ, on the other hand, is not issued where the
(a) detention is lawful,
(b) the proceeding is for contempt of a legislature or a court,
(c) detention is by a competent court, and
(d) detention is outside the jurisdiction of the court.
57
MISSION SHOURYA PSI TEST SERIES -2024

65. A man purchased wheat worth Rs. 400. He sold 3/4 at a loss of 10% and the reminder at a gain of
10%. On the whole he gets:
a) loss of 6%
b) gain of 5%
c) loss of 19%
d) loss of 5%
D. loss of 5%
C.P of 3/4th = Rs. (3/4 x 400) = Rs. 300,
C.P of 1/4th = Rs. 100
Therefore,
Total S.P = (90% of Rs. 300 + 110% of Rs. 100) = Rs. 380
Loss = (20/400 x 100) % = 5%

66. The average of runs of a cricket player of 10 innings was 32. How many runs must he make in his
next innings so as to increase his average of runs by 4?
a) 76
b) 79
c) 85
d) 87
A. 76
Average = total runs / no.of innings = 32
So, total = Average x no.of innings = 32 x 10 = 320.
Now increase in avg = 4runs. So, new avg = 32+4 = 36runs
Total runs = new avg x new no. of innings = 36 x 11 = 396
Runs made in the 11th inning = 396 - 320 = 76

67. In a 500 metres race, B starts 45 metres ahead of A, but A wins the race while B is still 35 metres
behind. What is the ratio of the speeds of A to B assuming that both start at the same time?
a) 25:21
b) 25:20
c) 5:3
d) 5:7
A.25: 21
Total distance = 500m
Distance covered by A= 500m
Distance covered by B= 500 – 45 – 35
= 500 – 80 = 420
(Speed of A/Speed of B) = (distance of A/distance of B)
500/420 = 25/21.

68. Out of 130 students appearing in an examination, 62 failed in English, 52 failed in Mathematics,
whereas 24 failed in both English and Mathematics. The number of students who passed finally is
a) 40
58
MISSION SHOURYA PSI TEST SERIES -2024
b) 50
c) 55
d) 60
A.40
Total students = 130
Total students failed = (62 + 52) – 24
= 114 – 24
= 90
Number of students passed finally= 130 – 90 = 40
69. The price of rice is increased by 20 percent and a person decrease his consumption by 15 percent,
so his expenditure on rice -?
a) increase by 2 percent
b) increase by 4 percent
c) decrease by 2 percent
d) decrease by 4 percent
A. increase by 2 percent
Let initial price of rice – 100
and new price of rice – 120
Suppose initial consumption is 100kg and new consumption is 85kg
Initial expenditure = 10000
New expenditure = 10200 (200/10000)*100 = 2 percent increase.

70. Abhay is taller than Vijay but shorter than Chandan, who is shorter than Deepak. Who is the
tallest among them?
a) Abhay
b) Bijay
c) Chandan
d) Deepak
D.Deepak
Abhay is taller than Vijay but shorter than Chandan
Chandan>Abhay>Vijay
Now Chandan is shorter than Deepak
Deepak>Chandan>Abhay>Vijay
Clearly, Deepak is the tallest among all.

71. How many four digit numbers can be formed by using the digits 0, 2, 4, 6, 7 if repetition of digits is
allowed?
a) 625
b) 96
c) 500
d) 36
C.500
Total digits = 5
First place can be filled up by using only one of 4 digits (except 0, since 0 at the first place is meaningless).
Second place can be filled up by using all the five digits (as repetition is allowed).
Similarly, third and fourth place can be filled up by using all the five digits.
Total numbers = 4 × 5 × 5 × 5 = 500

59
MISSION SHOURYA PSI TEST SERIES -2024
72. In how many different ways can the letters of the word „DRASTIC‟ be arranged in such a way
that the vowels always come together?
a) 720
b) 360
c) 1440
d) 540
C.1440
Total letters = D, R, A, S, T, I, C (7)
Total vowels = A, I (2)
∴ Required no. of ways = 6! × 2!
= 1440

73. Match the following


List 1(Countries) List 2 (Rockets)
1. Russia A. H-2
2. European Union B. Falcon
3. Japan C. Ariane
4. America D. Proton
Codes
a) 1-D, 2-C, 3-A, 4-B
b) 1-B, 2-A, 3-C, 4-D
c) 1-D, 2-A, 3-C, 4-B
d) 1-B, 2-C, 3-A, 4-D
A)1-D, 2-C, 3-A, 4-B
 Russia - Proton
 European Union - Ariane
 Japan - H-2
 America - Falcon

74. The main reason of the conflict between NawabSirajuddaula and the East India Company was
that?
a) The British opposed the succession of Sirajuddaula.
b The British misused the Dustaq (free duty passes)
c) The British attacked the French settlement of Chandranagar.
d) The incident of black hole had occurred.
Answer; B. The British misused the Dustaq (free duty passes)
Explination
 Mirza Muhammad Siraj-ud-Daulah, also known as Siraj-ud-Daulah or Siraj ud-Daula, was
Bengal's last independent Nawab. In 1740, he established Nizamat Imambara in Murshidabad, West
Bengal. The end of his reign signalled the beginning of the East India Company's dominion over
Bengal and, ultimately, practically the whole Indian subcontinent. At the age of 23, Siraj succeeded his
maternal grandfather, Alivardi Khan, as Nawab of Bengal in April 1756.
Siraj ud Daula – Reign
 During this time, the British East India Company was expanding its power in the Indian
subcontinent, notably in Bengal; Siraj quickly began to detest the East India Company's politico-
military presence in Bengal.
 He was particularly enraged by the Company's apparent involvement in, and sponsorship of, a coup plot
against him by some members of his own court.

60
MISSION SHOURYA PSI TEST SERIES -2024
 His allegations against the firm were essentially threefold.
 First, they strengthened the fortifications around Fort William without any intimation or approval;
Second, they grossly abused trade privileges granted to them by Mughal rulers – resulting in heavy
losses of customs duties for the government;
 Third, they provided shelter to some of his officers, such as Krishnadas, son of Rajballav, who
fled Dhaka after misappropriating government funds.
 The British success in the Carnatic wars has made Siraj-Ud-Daula fearful about the British increasing
influence in India.
 The Company's officials abused their trading rights in a way that harmed the nawab's finances.
 The British strengthened Fort Williams without Nawab's approval, infuriating him even more.
 When the East India Company began bolstering military power at Fort William in Calcutta, Siraj
ud-Daulah ordered a halt. Because the Company disobeyed his orders, Siraj responded by seizing
Calcutta (for a short time renamed Alinagar) from the British in June 1756.
 He marched to Fort Williams, grabbed 146 Britishers, and imprisoned them in a cramped chamber,
resulting in the deaths of 123 Britishers.
 This is known as the "Black Hole Tragedy of Calcutta."
 Siraj ud-Daulah was betrayed by Mir Jafar, the leader of the Nawab's army, and lost the Battle of
Plassey on June 23, 1757.
 The East India Company's soldiers, led by Robert Clive, invaded Bengal, seizing control of the
administration.
 Mohammad Ali Beg executed Siraj-ud-Daulah on 2 July 1757 on instructions from Mir Miran, son of
Mir Jafar, at Namak Haram Deorhi as part of the agreement between Mir Jafar and the British East
India Company
 The grave of Siraj-ud-Daulah is located in Khushbagh, Murshidabad.
 The end of Siraj ud-rule Daulah's also signified the end of Bengali autonomy and the establishment of
British control in India.
Battle of Plassey
 The Battle of Plassey took place on June 23, 1757, in the Plassey district of West Bengal.
 The British East India Company, led by Robert Clive, fought this battle against the Nawab of
Bengal, Siraj-ud-daula.
 The French forces took part in this battle as well, fighting with Siraj-ud-daula against Robert Clive.
 The Nawab of Bengal's army of 50,000 warriors, with French forces on their side, vastly overpowered
the British army of roughly 3000 soldiers.
 However, Robert Clive's conspiracy and subsequent betrayal by Mir Jafar, Rai Durlabh, and others led
to the downfall of Nawab of Bengal Siraj-ud-daula in the Battle of Plassey.
 The Fight of Plassey is seen as a momentous battle for the British and a turning point in Indian history
in their favour.
 It established the British military and political dominance in Bengal.
 The battle of Plassey is regarded by most historians as the crucial event, with origins in British
control and governance over India.

75. Arrange the following events chronologically:


1. Cripp’s Mission
2. Cabinet Mission
3. Quit India Movement
4. Individual Satyagraha
5. RIN Mutiny
Select the correct answer from the codes given below:
a) 1– 4 – 2 – 3 – 5
b) 4 – 1 – 3 – 5 – 2
c) 4 – 1 – 3 – 2 – 5

61
MISSION SHOURYA PSI TEST SERIES -2024
d) 4 – 1 – 2 – 3 – 5
b. 4 – 1 – 3 – 5 – 2
 Individual Satyagraha arose directly as a result of the August Offer. The British made the August offer during
a critical period of the war in 1940. The August Offer was rejected by both Congress and the Muslim League. It
began with the Civil Disobedience Movement, but M.K Gandhi focused on Individual Satyagraha.
Involvement in Individual Satyagraha
 Vinoba Bhave was the first person chosen to offer an individual satyagraha. On October 17, 1940, he
began his campaign at Paunar, only five miles from Wardha.
 In a speech, he asked the people not to participate in the Government's war effort
 Vinoba Bhave delivered anti-war speeches in Surgaon, Saloo, and Deoli for three days in a row, from
October 18 to 20, 1940.
 On October 25, 1940, the government issued orders prohibiting all anti-War propaganda in order to
combat Congress propaganda.
 Gandhiji saw this as a challenge and chose Jawaharlal Nehru as the second Satyagrahi after Vinoba
Bhave.
 Brahma Dutt, an ashram inmate, was the third person chosen to offer an individual satyagraha.
 The Cripps Mission was sent by the British government to India in March 1942 to obtain Indian
cooperation for the British war efforts in the 2nd World War. It was headed by Sir Richard Stafford
Cripps, a labour minister in Winston Churchill’s coalition government in Britain.
 Quit India Movement;Also known as August Movement or August Kranti, the Quit India Movement
was a clarion call given by Mahatma Gandhi on 08 August 1942 from the Bombay session of the All-
India Congress Committee in Mumbai. It is a part of the Civil Disobedience Movement launched by
Mahatma Gandhi to end British rule in India.
 On 8th Aug 2020, India completed 78 years of the Quit India Movement also known as August Kranti
 Seventy-four years ago on February 18, 1946, some 1,100 Indian sailors or “ratings” of the HMIS
Talwar and the Royal Indian Navy (RIN) Signal School in Bombay declared a hunger strike, triggered
by the conditions and treatment of Indians in the Navy.
 The Cabinet Mission was a high-powered mission sent to India by the Atlee government in February
1946. Three British cabinet members served on the mission: (Pethick Lawrence, Secretary of State for
India; Stafford Cripps, President of the Board of Trade; and A.V. Alexander, First Lord of the
Admiralty) to find ways and means for a negotiated, peaceful transfer of power to India. The mission's
chairman was Pethick Lawrence

76. Which of the following protects the body against infectious disease and foreign invaders?
a) Leukocytes
b) Red blood cells
c) Endoplasmic reticulum (ER)
d) Golgi apparatus
a. Leukocytes
Red Blood Cells (RBC)
 Erythrocytes or red blood cells (RBC) are the most abundant of all the cells in blood.
 A healthy adult man has, on an average, 5 million to 5.5 million of RBCs mm-3 of blood.
 RBCs are formed in the red bone marrow in the adults. RBCs are devoid of nucleus in most of the
mammals and are biconcave in shape.
 They have a red coloured, iron containing complex protein called haemoglobin, hence the colour and
name of these cells.
 RBCs have an average life span of 120 days after which they are destroyed in the spleen (graveyard of
RBCs).
 White Blood Cells (WBC)
 Leucocytes are also known as white blood cells (WBC) as they are colorless due to the lack of
haemoglobin. They are nucleated and are relatively lesser in number which averages 6000-8000 mm-3
of blood. Leucocytes are generally short lived. We have two main categories of WBCs –
62
MISSION SHOURYA PSI TEST SERIES -2024
granulocytes and agranulocytes.
 Neutrophils, eosinophils and basophils are different types of granulocytes, while lymphocytes and
monocytes are the agranulocytes.
 Neutrophils are the most abundant cells (60-65 per cent) of the total WBCs and basophils are the least
(0.5-1 per cent) among them.
 Neutrophils and monocytes (6-8 per cent) are phagocytic cells which destroy foreign organisms
entering the body.
 Basophils secrete histamine, serotonin, heparin, etc., and are involved in inflammatory reactions.
 Eosinophils (2-3 per cent) resist infections and are also associated with allergic reactions.
 Lymphocytes (20-25 per cent) are of two major types – ‘B’ and ‘T’ forms. Both B and T lymphocytes
are responsible for immune responses of the body.
 Endoplasmic Reticulum is a network of membranous tubules present within the cytoplasm of a
eukaryotic cell
 Golgi apparatus is the membrane-bound organelle of eukaryotic cells which are mainly involved in
transporting synthesized macromolecules within the cell.

77. The cost price of 15 chocolates is equal to the selling price of 30 chocolates. Find the profit/loss
percentage.
a) 20% Profit
b) 30% Loss
c) 50% Loss
d) 20% Loss
Answer; c. 50% Loss
15×CP=30×SP
⇒SP/CP=15/30
SP/CP−1=15/30−1
⇒SP−CP/CP=15−30/20
=Loss=15/30
Loss percentage =15/30×100=50%

78. The first speaker against whom a motion of non-confidence was moved in the LokSabha was ?
a) Balram Jakhar
b) G.V.Mavalankar
c) Hukum Singh
d) K.M.Hegde
b. G.V.Mavalankar
 On 27 November 1888, Ganesh Vasudev Mavalankar, the first Speaker of the Lok Sabha was born
into a Marathi family in Baroda.
 Mavalankar again was made the Speaker of the Constituent Assembly (Legislative) on November 17,
1947.
 He was made the speaker of the Provisional Parliament which was formed on November 26, 1949. Until
1952, he occupied this chair.
 The first Lok Sabha was constituted in 1952. Mavalankar was the obvious choice of Speaker. He
defeated the opponent to the post of the speaker by a comfortable margin.
 As Speaker, he made several innovations to the rules of the house. He was largely responsible for the
‘Question Hour’session being a regular and significant aspect of the parliament. It was again
Mavalankar who initiated the discussion on ‘President’s Address’ and ‘Motion of Thanks’.
 He also took initiatives to start committees like the Rules Committee, the Business Advisory
Committee, the Committee of Privileges, the Committee on Subordinate Legislation, the Committee on
Private Members’ Bills and Resolutions, the Committee on Absence of Members from the Sittings of
the House, the Committee on Government Assurances, Joint Committee on Salaries and Allowances of
Members of Parliament, etc.
 In fact, Mavalankar created and established many rules for procedure and conduct of the House which
63
MISSION SHOURYA PSI TEST SERIES -2024
are followed to this day.
 He was nicknamed the ‘Father of the Lok Sabha’ by Jawaharlal Nehru. He was also fondly called
‘Dadasaheb’.
 As a Speaker, he was exemplary due to his non-partisanship despite his affiliations to the INC.
Throughout his tenure as speaker, he did not take an active part in party politics.
 In 1950, he represented India at the opening ceremony of the new House of Commons in London.
 Mavalankar had a cardiac arrest in January 1956 and a month later on 27 February 1956, he passed
away in Ahmedabad. He had been serving as the Lok Sabha Speaker when he died.

79. The foreign traveler who visited and described Vijayanagar soon after its fall in the battle of
Talikota, was
a) Abdur Razzak
b) Ceasar Frederick
c) Nicolo Conti
d) Nuniz
Answer; B. Ceasar Frederick
 Cesare Frederici, an Italian traveller who spent seven months at Vijayanagara in 1567, two years after
the city was sacked, suggests that the capital was only partly destroyed and that Tirumala of the Aravidu
dynasty intended to re-establish the Vijayanagara capital there. This attempt turned out to be
unsuccessful and the city was eventually abandoned for good.
Battle Of Talikota (1565)
 The Battle of Talikota (23 January 1565) was a watershed battle fought between the Vijayanagara
Empire and the Deccan sultanates. The battle took place at Talikota, today a town in northern
Karnataka, about 80 kilometres (50 mi) to the southeast from the city of Bijapur. The treacherous defeat
of Vijayanagara Empire, followed subsequent destruction and looting which became short lived before
the successors of Rama Raya.
 The Vijayanagara army was winning the battle, state Hermann Kulke and Dietmar Rothermund, but
suddenly two Muslim generals of the Vijayanagara army switched sides and turned their loyalty to the
united Sultanates. They captured Rama Raya and beheaded him on the spot, with Sultan Hussain on the
Sultanates side joining them. Last ruler of Vijayanagara kingdom was Sri ranga III.

80. Match List – 1 (Ancient places) with List-2 (their Present location) and select the correct answer.
List-1 List - 2
A. Rupar 1. Sind
B. Lothal 2. Rajasthan
C. Kalibangan 3. Ahmedabad
D. KotDiji 4. Chandigarh
Codes
A B C D
a) 4 3 2 1
b) 1 2 3 4
c) 2 3 4 1
d) 3 4 2 1
Answer; A 4 3 2 1
 The Archaeological Museum Ropar is situated on the bank of River Sutlej along the Ghaggar-Hakra
beds. It is about 40 km north east of Chandigarh and it lies on Rupnagar – Chandigarh highway. In
1998, the museum was opened to the public. The museum showcases the archaeological remains of an
excavated site near Ropar.
Lothal
 R.Rao in 1953; Gujarat on Bhogva river near Gulf of Cambay
64
MISSION SHOURYA PSI TEST SERIES -2024
 First manmade port
 Dockyard
 Rice husk
 Fire altars
 Chess playing
Kalibangan
 Ghose in 1953 Rajasthan on the bank of Ghaggar river
 Fire altar
 Camel bones
 Wooden plough
The ancient site at Kot Diji (Sindhi:
 Urdu: was the forerunner of the Indus Civilization. The occupation of this site is attested already at 3300
BCE. The remains consist of two parts; the citadel area on high ground (about 12 m [39 ft])), and outer
area. The Pakistan Department of Archaeology excavated at Kot Diji in 1955 and 1957.
 Located about 45 km (28 mi)) south of Khairpur in the province of Sindh, Pakistan, it is on the east
bank of the Indus opposite Mohenjo-daro.
 The site is situated at the foot of the Rohri Hills where a fort (Kot Diji Fort) was built around 1790 by
the Talpur dynasty ruler of Upper Sindh, Mir Suhrab, who reigned from 1783 to 1830 AD. This fort
built on the ridge of a steep narrow hill is well-preserved.

81. With reference to the cultural history of India, the “Kirtana-ghosha” was composed by:
a) Shankaradeva
b) Mirabai
c) Kabir
d) Raidas
Answer; a. Shankaradeva
Sankardev (1449 - 1568)
 According to traditional accounts, Sankardev, then known as Sankara, was born in 1449 CE in
Alipukhuri near Bordowa in the present-day Nagaon region of Assam.
 His father, Kusumvar Bhuyan, was the Shiromani (head) of the region's Baro-Bhuyans clans (the
Baro-Bhuyans were independent landlords in Assam).
 He is largely recognized for inventing new types of music (Borgeet), theatrical performance (Ankia
Naat, Bhaona), dance (Sattriya), and literary language (Brajavali).
 In addition, he left a large literary corpus that included trans-created scriptures, poetry, and theological
writings written in Sanskrit, Assamese, and Brajavali.
 The Bhagavatic religious movement he founded, Ekasarana Dharma, is also known as the Neo-
Vaishnavite movement.
 Influenced two medieval kingdoms - Koch and the Ahom kingdoms and the assembly of devotees he
founded, evolved over time into monastic centers known as Sattras, which remain important socio-
religious institutions in Assam and, to a lesser extent, North Bengal.
 Sankardev influenced the Bhakti movement in Assam in the same way as Guru Nanak, Ramananda,
Namdev, Kabir, Basava, and Chaitanya Mahaprabhu inspired it throughout the Indian subcontinent.
 His ideas were approved by the Matak Kingdom, which was formed by Bharat Singha and
consolidated by Sarbananda Singha in the late 18th century.

82. With respect to the Tourism taglines of the Indian States, which one of the following pair is not
correctly matched?
a) Madhya Pradesh – The Heart of Incredible India
b) Manipur – Jewel of Incredible India
65
MISSION SHOURYA PSI TEST SERIES -2024
c) Rajasthan – The Incredible State of India
d) Kerala – One State Many Worlds.
Answer;D. Kerala – One State Many Worlds.
States and Their Tourism Tagline
State Tourism Tagline

Andhra Pradesh The Essence of Incredible India


Arunachal Pradesh The Land of Dawnlit Mountains
Assam The Awesome Assam
Bihar Blissful Bihar
Chhattisgarh Full of Surprises
Goa A Perfect Holiday Destination
Haryana A pioneer in Highway Tourism
Himachal Pradesh Unforgettable Himachal
Jammu and Kashmir Chalo Kashmir
Jharkhand A New Experience
Karnataka One state Many Worlds
Kerala God’s Own Country
Madhya Pradesh The Heart of Incredible India
Maharashtra Unlimited
Manipur Jewel of Incredible India
Meghalaya Half Way To Heaven
Nagaland Land of Festivals
Orissa The Soul of Incredible India
Punjab India Begins Here
Rajasthan The Incredible State of India
Sikkim Small But Beautiful
Telangana It’s all in it
Tamil Nadu Enchanting Tamil Nadu
Tripura Visit Agartala
Uttarpradesh Amazing Heritage Grand Experiences
Uttarakhand Exploring Uttarakhand
West Bengal Beautiful Bengal

83. With reference to official languages consider the following statements


66
MISSION SHOURYA PSI TEST SERIES -2024
1. Tulu was recently included in the Eight Schedule of the Constitution.
2. It is the first tribal language to be included in the Eight Schedule.
Which of the statements given above is/are correct?
a) 1 only
b) 2 only
c) Both 1 and 2
d) Neither 1 nor 2
d. Neither 1 nor 2
Official Language or Languages of a State
 Part XVII of the Indian Constitution deals with the official language in Articles 343 to 351.
 Article 345 of the Constitution says “the Legislature of a State may by law adopt any one or more of the
languages in use in the State or Hindi as the language or languages to be used for all or any of the
official purposes of that State”.
Eighth Schedule of the Constitution
 The Constitutional provisions relating to the Eighth Schedule occur in Article 344 (1) and 351 of the
Constitution.
The languages listed under Eighth Schedule are:
 (1) Assamese, (2) Bengali, (3) Gujarati, (4) Hindi, (5) Kannada, (6) Kashmiri, (7) Konkani, (8)
Malayalam, (9) Manipuri, (10) Marathi, (11) Nepali, (12) Oriya, (13) Punjabi, (14) Sanskrit, (15)
Sindhi, (16) Tamil, (17) Telugu, (18) Urdu (19) Bodo, (20) Santhali, (21) Maithili and (22) Dogri.
 Languages are added through Constitutional Amendments.
about 'Tulu' Language:
 Tulu is a Dravidian language spoken mainly in two coastal districts Dakshina Kannada and Udupi
of Karnataka and Kasaragod district of Kerala.
 This region is informally known as Tulu Nadu. There has been a demand for separate statehood for
Tulu Nadu.

67
MISSION SHOURYA PSI TEST SERIES -2024

84. Consider the following statements:


1. Karnataka shares its boundaries with five India states.
2. ‘Malnad’ is the hilly region of Karnataka, which comprises the Western Ghats.
Which of the statements given above is/are correct?
a) 1 only
b) 2 only
c) Both 1 and 2
d) Neither 1 nor 2
b. 2 only

68
MISSION SHOURYA PSI TEST SERIES -2024

 Malenadu (mǝ-le-nah-doo; simplified as Malnad) is a region in the state of Karnataka in


India. Malenadu covers the western and eastern slopes of the Western Ghats or Sahyadri
mountain range, and is roughly 100 kilometers in width

85. Match the acids (List I) with the sources (List II)
List I (Acids) List II (Sources)
A. Tartaric acid I. Red ants
B. Formic acid II. Grapes
C. Uric acid III. Apples
D. Maleic acid IV. Urine of mammals
Select the code for the correct answer from the options given below.
A B C D
a. 1 2 3 4
b. 2 1 4 3
c. 2 1 3 4
d. 1 2 4 3
b. 2 -1 - 4 - 3

69
MISSION SHOURYA PSI TEST SERIES -2024

86. Pneumatophores, an adaptation found in Mangroves are


a) Respiratory roots
b) Props roots
c) Germination of seeds while on tree
d) None of the above
a. Respiratory roots
 Mangrove species have specialized above ground roots called breathing roots or pneumatophores.
Mangroves
 Mangroves are plants that grow in inter-tidal areas. They are halophytes (‘salt-loving”). Most plants can not
survive in salty conditions.
 Mangroves live in a unique environment-between land and sea.
 Mangroves represent a characteristic littoral (near the seashore) forest ecosystem.
 These are mostly evergreen forests that grow in sheltered low-lying coasts, estuaries, mudflats, tidal
creeks backwaters (coastal waters held back on land), marshes, and lagoons of tropical and
subtropical regions.
 Mangroves grow below the high water level of spring tides.
 The best locations are where abundant silt is brought down by rivers or on the backshore of accreting
sandy beaches.
 Mangroves are highly productive ecosystems, and the trees may vary in height from 8 to 20 m. They
protect the shoreline from the effect of cyclones and tsunamis.
 They are breeding and spawning ground for many commercially important fishes.
 Since mangroves are located between the land and sea, they represent the best example of the ecotone.
 Mangroves are shrubs or small trees that grow in coastal saline or brackish water.
 Mangroves are salt-tolerant trees, also called halophytes, and are adapted to harsh coastal conditions.
 Mangrove vegetation facilitates more water loss. Leaves are thick and contain salt-secreting glands.
70
MISSION SHOURYA PSI TEST SERIES -2024
Some block absorption of salt at their roots itself.
 They contain a complex salt filtration system and complex root system to cope with saltwater
immersion and wave action.
 They are adapted to the low oxygen (anoxic) conditions of waterlogged mud.
 They produce pneumatophores (blind roots) to overcome the respiration problem in anaerobic
soil conditions.
 Mangroves occur worldwide in the tropics and subtropics, mainly between latitudes 25° N and 25° S.
 They require high solar radiation to filter saline water through their roots. This explains why
mangroves are confined to only tropical and subtropical coastal waters.
 Mangroves occur in a variety of configurations. Some species (e.g. Rhizophora) send arching prop
roots down into the water.
 While others (e.g. Avicennia) send vertical “Pneumatophores” or air roots up from the mud.
 Adventitious roots that emerged from the main trunk of a tree above ground level are called stilt
roots.

87. Which of the following pairs is incorrectly matched?


a) Forest Act – 1928
b) Wildlife (Protection) Act – 1972
c) National Forest Policy – 1988
d) Water (Prevention and Control of Pollution)Act- 1974.
a. Forest Act – 1928
 Forest Act – 1927
 Wildlife (Protection) Act – 1972
 National Forest Policy – 1988
 Water (Prevention and Control of Pollution)Act- 1974.
 Indian Forest Act of 1927:This Act impacted the life of forest-dependent communities. The penalties
and procedures given in this Act aimed to extend the state’s control over forests as well as diminishing
the status of people’s rights to forest use.
 The village communities were alienated from their age-old symbiotic association with forests. Further
71
MISSION SHOURYA PSI TEST SERIES -2024
amendments were also made to restrain the local use of forests mainly by forest-dependent
communities.
 It was enacted to make forest laws more effective and to improve the previous forest laws.
Objective
 To consolidate all the previous laws regarding forests.
 To give the Government the power to create different classes of forests for their effective usage for the
colonial purpose.
 To regulate movement and transit of forest produce, and duty leviable on timber and other forest
produce.
 To define the procedure to be followed for declaring an area as Reserved Forest, Protected Forest or
Village Forest.
 To define forest offences acts prohibited inside the Reserved Forest, and penalties leviable on the
violation.
 To make conservation of forests and wildlife more accountable.
Types of Forests
 Reserved Forests: Reserve forests are the most restricted forests and are constituted by the State
Government on any forest land or wasteland which is the property of the Government.
 In reserved forests, local people are prohibited, unless specifically allowed by a Forest Officer in the
course of the settlement.
 Protected Forests: The State Government is empowered to constitute any land other than reserved
forests as protected forests over which the Government has proprietary rights and the power to issue
rules regarding the use of such forests.
 This power has been used to establish State control over trees, whose timber, fruit or other non-wood
products have revenue-raising potential.
 Village forest:Village forests are the one in which the State Government may assign to ‘any village
community the rights of Government to or over any land which has been constituted a reserved forest’.
Degree of protection
 Reserved forests > Protected forests > Village forests
Wildlife (Protection) Act, 1972
Constitutional Provisions for Wildlife:
 The 42nd Amendment Act, 1976, Forests and Protectionof Wild Animals and Birds was transferred from State
to Concurrent List.
 Article 51 A (g) of the Constitution states that it shallbe the fundamental duty of every citizen to protect and
improve the natural environment including forests and Wildlife.
 Article 48 A in the Directive Principles of State policy, mandates that the State shall endeavor to protect and
improve the environment and to safeguard the forests and wildlife of the country.
National Forest Policy – 1988
 Maintenance of environmental stability through preservation and restoration of ecological balance.
 Conservation of forest as natural heritage.Check on soil erosion and denudation in catchment areas of rivers,
lakes and reservoirs.
 Check on extension of sand dunes in the desert areas of Rajasthan and along coastal tracts.
 Substantial increase in forest cover through massive afforestation and social forestry schemes.
 Steps to meet requirement of fuel wood, fodder, minor forest produce and soil timber of rural and tribal
populations.
 Increase in productivity of forest to meet the national needEncouragement of efficient utilization of forest
produce and optimum substitution of wood.
 Steps to create awareness between peoples and achieve the objectives with minimizing the pressure on existing
forests.
 Involvement of people in forest management under Joint forest Management.
Water (Prevention and Control of Pollution) Act, 1974

72
MISSION SHOURYA PSI TEST SERIES -2024
 The Water (Prevention and Control of Pollution) Act was enacted in 1974 to provide for the prevention and
control of water pollution, and for the maintaining or restoring of wholesomeness of water in the country.
 The Act was amended in 1988. The Water (Prevention and Control of Pollution) Cess Act was enacted in 1977,
to provide for the levy and collection of a cess on water consumed by persons operating and carrying on certain
types of industrial activities.
 This cess is collected with a view to augment the resources of the Central Board and the State Boards for the
prevention and control of water pollution constituted under the Water (Prevention and Control of Pollution) Act,
1974. The Act was last amended in 2003.
Central Pollution Control Board (CPCB)
 The Central Pollution Control Board (CPCB), statutory organisation, was constituted in September, 1974 under
the Water (Prevention and Control of Pollution) Act, 1974. Further, CPCB was entrusted with the powers and
functions under the Air (Prevention and Control of Pollution) Act, 1981

88. Which of the following tribes are commonly matched with their prominent state of evidence
1. Irulas : Kerala
2. Jaintia : Meghalaya
3. Chenchu : Arunachal Pradesh
Select the answer using code
a) 1 and 2 only
b) 1 and 3 only
c) 2 and 3 only
d) 1, 2 and 3
a. 1 and 2 only
State Tribes
Andhra Pradesh Andh And Sadhu Andh, Bhil, Bhaghata, Dhulia,rona, Kolam, Gond, Thoti, Goundu,
Kammara, Savaras, Dabba Yerukula, Sugalis, Nakkala, Pardhan, Gadabas, Chenchus A.k.a
Chenchawar, Kattunayakan, Jatapus, Manna Dhora
Arunachal Singpho, Monpa, Abor, Sherdukpen, Galo, Apatanis
Pradesh
Assam Khasis, Chakma, Dimasa, Gangte, Garos, Hajong, Chutiya
Bihar Gond, Birjia, Asur, Savar, Parhaiya, Chero, Birhor, Santhals, Baiga
Chhattisgarh Nagasia, Biar, Khond, Agariya, Bhattra, Mawasi, Bhaina,
Goa Varli, Dubia, Siddi, Dhodia, Naikda
Gujarat Patelia, Bhil, Dhodia, Bamcha, Barda, Paradhi, Charan, Gamta
Himachal Swangal, Gujjars, Lahaulas, Khas, Pangwala, Lamba, Gaddis
Pradesh
Jammu and Balti, Garra, Sippi, Bakarwal, Mon, Gaddi, Purigpa, Beda
Kashmir
Jharkhand Gonds, Birhors, Savar, Mundas, Santhals, Khaira, Bhumji
Karnataka Gond, Patelia, Barda, Yerava, Bhil, Koraga, Adiyan, Iruliga,
Kerala Malai, Aarayan, Arandan, Uralis, Kurumbas, Arandan, Eranvallan
Madhya Kharia, Bhils, Murias, Birhors, Baigas, Katkari, Kol, Bharia, Khond, Gonds,
Pradesh
Maharashtra Warlis, Khond, Bhaina, Katkari, Bhunjia, Rathawa, Dhodia.
Manipur Thadou, Aimol, Maram, Paite, Chiru, Purum, Kuki, Monsang, Angami
Meghalaya Pawai, Chakma, Raba, Hajong, Lakher, Garos, Jaintias Khasis
Mizoram Dimasa, Raba, Chakma, Lakher, Khasi, Synteng, Kuki, Pawai.
Nagaland Nagas, Angami, Sema, Garo, Kuki, Kachari, Mikir
Odisha Gadaba, Ghara, Kharia, Khond, Matya, Oraons, Rajuar, Santhals.
Rajasthan Bhils, Damaria, Dhanka, Meenas(Minas), Patelia, Sahariya.
Sikkim Bhutia, Khas, Lepchas.
Tamil Nadu Adiyan, Aranadan, Eravallan, Irular, Kadar, Kanikar, Kotas, Todas.
73
MISSION SHOURYA PSI TEST SERIES -2024
Telangana Chenchus.
Tripura Bhil, Bhutia, Chaimal, Chakma, Halam, Khasia, Lushai, Mizel, Namte.
Uttarakhand Bhotias, Buksa, Jannsari, Khas, Raji, Tharu.
Uttar Pradesh Bhotia, Buksa, Jaunsari, Kol, Raji, Tharu.
West Bengal Asur, Khond, Hajong, Ho, Parhaiya, Rabha, Santhals, Savar.
Andaman and Oraons, Onges, Sentinelese, Shompens.
Nicobar
Little Andaman Jarawa
North-East Abhors, Chang, Galaong, Mishimi, Singpho, Wancho.

89. Match the following Dams with river across which it has built,
Dam River
A. Supa 1. Kali
B. Hidkal 2. Ghataprabha
C. Manidam 3. Varahi
Select the correct answer using code,
a) A – 2, B – 1, C – 3
b) A – 1, B – 3, C – 2
c) A – 1, B – 2, C – 3
Ans: c. A – 1, B – 2, C – 3

Dam/Reservoir River Location


Almatti Dam Krishna Basavana Bagevadi taluk, Bijapur district
Basava Sagara Krishna Narayanapur, Hunasagi taluk, Yadgir district
Dam(Narayanpur Dam)
Raja Lakhamagowda dam Ghataprabha Hidkal, Hukkeri taluk, Belgaum district
Renuka Sagara Dam Malaprabha Navilutheertha, Saundatti taluk, Belgaum district
Vani Vilasa Sagara Vedavathi Marikanive, Hiriyur Taluk, Chitradurga district
Upper Tunga Dam Tunga Gajanur, Shimoga
Bhadra Dam Bhadra Lakkavalli,
Tarikere taluk, Chikmagaluru district
Tungabhadra Dam Tungabhadra Hospet, Vijayanagara district
Hemavathi Reservoir Hemavathi Gorur, Hassan district
Kabini Reservoir Kabini Heggadadevanakote Taluk, Mysore
Harangi Reservoir Harangi Hudgur, Somawarapete taluk, Kodagu district
Krishna Raja Sagara Dam Kaveri Mandya
Linganamakki Dam Sharavathi Linganamakki, Sagara Taluk, Shimoga district
Chakra Reservoir Chakra Chakra Nagar, Hosanagar Taluk, Shimoga
Savehaklu Reservoir Chakra Chakra Nagar, Hosanagar Taluk, Shimoga
Mani Reservoir Varahi Hummadagallu, Hosanagar Taluk, Shimoga
Supa Dam Kali Ganesha Gudi, Joida Taluk, Uttara Kannada district
Kodasalli Dam Kali Kodasalli, Joida Taluk, Uttara Kannada district
Kadra Dam Kali Virje Karwar Taluk, Uttara Kannada district
Shanti Sagara Haridra Kerebilchi, Channagiri taluk, Davanagere district
Karanja Dam Karanja Byalahalli, Bhalki taluk, Bidar district

74
MISSION SHOURYA PSI TEST SERIES -2024
90. Arrange of the following states based on the length of their coast line.
1. Karnataka
2. Gujarat
3. Andhra Pradesh
4. Odisha
Select the correct answer using code below.
a) 3 – 1 – 4 – 2
b) 2 – 3 – 4 – 1
c) 2 – 3 – 1 – 4
d) 3 – 2 – 4 – 1
Ans: b. 2 – 3 – 4 – 1

Coastal States of India


 India has a coastline of 7516.6 km-- 5422.6 km of mainland coastline and 2094 km of island territories.
Indian coastline touches nine states-- Gujarat, Maharashtra, Goa, Karnataka, Kerala, Tamil Nadu,
Andhra Pradesh, Odisha, West Bengal and two union territories-- Daman and Diu and Puducherry. The
two island territories of India are-- Andaman and Nicobar Islands in the Bay of Bengal and
Lakshadweep Islands in the Arabian Sea.
Coastal Data

Length of Coastline 7516.6 km

Total Land Area 3,287,263 km2

Area of the continental shelf 372,424 km2

Territorial sea (up to 12 nautical miles) 193,834 km2

Exclusive Economic Zone 2.02 x 106 million km2

Coastal Geomorphology (Mainland)

Sandy Beach 43%

Rocky Coast 11%

Muddy Flats 36%

75
MISSION SHOURYA PSI TEST SERIES -2024
Marshy Coast 10%
Coastline affected by erosion 1624.435 km mainland
132 (islands) (CPDAC)
Population of the Coastal States and UTs 560 million
Population of Island Territories 0.44 illion

91. Steel plants in India during 1960‟s were built in collaboration with foreign countries. Which of the
following is correctly matched?
Plant Country
1. Rourkela - German
2. Durgapur - British
3. Bhilai - USA
4. Bokaro - Russia
Select the correct answer using code,
a) 1, 2 and 3 only
b) 2, 3 and 4 only
c) 1, 2 and 4 only
d) 1, 3 only 4 only
c. 1, 2 and 4 only
 Bhilai And Bokaro SteelPlants were set up with Soviet collaboration. Rourkela Steel Plant was set up with
German collaboration.
 Durgapur Steel Plant was set up with the collaboration of the United Kingdom.
 Visakhapatnam Steel Plantis India's first shore-based integrated steel plant.

76
MISSION SHOURYA PSI TEST SERIES -2024
92. Arrange the following rivers in North South direction.
1. Sharavati
2. Aghanashini
3. Netravati
4. Kalinadi
Codes:
a) 1 – 4 – 3 – 2
b) 4 – 3 – 2 – 1
c) 1 – 3 – 2 – 1
d) 4 – 2 – 1 – 3
d. 4 – 2 – 1 – 3

93. Some Buddhist rock-cut caves are called Chaityas, while the others are called Viharas. What is the
difference between the two?
a) Vihara is a place of worship, while Chaitya is the dwelling place of the monks
b) Chaitya is a place of worship, while Vihara is the dwelling place of the monks.
c) Chaitya is the stupa at the far end of the cave, while Vihara is the hall axial to it.
d) There is no material difference between the two
B Chaitya is a place of worship, while Vihara is the dwelling place of the monks.
 Viharas were for the purpose of living, Chaityas were assemblies for the purpose of discussions.
Further, Chaityas were with Stupas, Viharas did not have stupas.
 Both early Chaityas and Viharas were made by woods and later stone-cut Chaityas and Viharas were
77
MISSION SHOURYA PSI TEST SERIES -2024
made. Chaitya was a rectangular prayer hall with a stupa placed in the centre, the purpose was prayer.
The Chaitya was divided into three parts, and had an apsidal ending, that is, a semicircular rear end, The
central part of the hall (also called the nave) was separated from the two aisles by two rows of pillars,
The chaityas also had polished interior walls, semicircular roofs and horse-shoe shaped windows called
the Chaitya windows. Viharas were the residences of the monks.

94. Match the list I with List II


List I List II
A. Bhabar I. Marshyland at the edge
of Alluvial plain
B. Tehari II. Kankar is the main
feature
C. Bhangar III. Dry valley with gravel
and unsorted sediments
D. Khader IV. The younger alluvion of
the flood plains.

Codes
a) A-3 B-4 C-2 D-1
b) A-2 B-1 C-4 D-3
c) A-2 B-4 C-1 D-3
d) A-3 B-1 C-2 D-4
D) A-3. B-1. C-2. D-4

95. Consider the International monetary Institues and Organaizations


1. International monetary fund (IMF)
2. World Bank
3. Asia development Bank
4. New Development Bank
Organise the institutes in chronological order on their Establishment.
a) 1, 2, 3, 4
b) 2, 1, 4, 3
c) 1, 2, 4, 3
78
MISSION SHOURYA PSI TEST SERIES -2024
d) 2, 1, 3, 4
Ans : d) 2, 1, 3, 4
IMF
 The IMF, also known as the Fund, was conceived at a UN conference in Bretton Woods, New
Hampshire, United States, in July 1944.
 The 44 countries at that conference sought to build a framework for economic cooperation to avoid a
repetition of the competitive devaluations that had contributed to the Great Depression of the 1930s.
 Countries were not eligible for membership in the International Bank for Reconstruction and
Development (IBRD) unless they were members of the IMF.
 IMF, as per Bretton Woods agreement to encourage international financial cooperation, introduced a
system of convertible currencies at fixed exchange rates, and replaced gold with the U.S. dollar (gold at
$35 per ounce) for official reserve.

World Bank
 About:
o It was created in 1944, as the International Bank for Reconstruction and Development (IBRD)
along with the IMF. The IBRD later became the World Bank.
o The World Bank Group is a unique global partnership of five institutions working for sustainable
solutions that reduce poverty and build shared prosperity in developing countries.
o The World Bank is one of the United Nations' specialized agencies.
 Members:
o It has 189 member countries.
o India is also a member country.

Asian Development Bank (ADB):


 It is a multilateral development bank established on 19th December 1966.
 Its primary mission is to "foster economic growth and cooperation" among countries in the Asia-
Pacific Region.
 Functions:
o ADB assists members and partners by providing loans, technical assistance, grants, and
equity investments to promote social and economic development.
o It also provides financing to certain private sector projects as well as public-private
partnerships.
o The ADB regularly facilitates policy dialogues and provides advisory services.
o They also use co-financing operations that tap official, commercial, and export credit
sources while providing assistance.
 Headquarters: Manila, Philippines.
 Members: From 31 members at its establishment in 1966, ADB has grown to encompass 68
members—of which 49 are from within Asia and the Pacific and 19 outside.
New Development Bank (NDB):
 NDB, formerly referred to as the BRICS Development Bank, is a multilateral development bank
established by the BRICS countries (Brazil, Russia, India, China and South Africa).
 Objective: Financing infrastructure and sustainable development projects in BRICS and other
emerging economies and developing countries.
 The idea of setting up NDB was first conceived in 2012 during BRICS Summit in New Delhi, India.
The Bank formally came into existence as a legal entity in 2015.
 Headquarters: Shanghai, China
 The first regional office of the NDB was setup in Johannesburg, South Africa. The second regional
office was established in 2019 in São Paulo, Brazil, followed by Moscow, Russia.
 Capital: The Bank has an initial authorized capital of 100 billion dollars and an initial subscribed
capital of 50 billion dollars.

79
MISSION SHOURYA PSI TEST SERIES -2024
 Membership in NDB is open to any member of the United Nations.

96. The sum of four consecutive even numbers A, B, C and D is 180. then the sum of next four
consecutive even numbers is?
a) 196
b) 204
c) 212
d) None of the above
C) 212
A + A + 2 + A + 4 + A + 6 = 180
4A + 12 = 180
A = 42.
Next four consecutive even numbers are 50 + 52 + 54 + 56 = 212

97. With reference to ‘Apna Chandrayaan’, consider the following statements:


1. It is launched by the Ministry of Education and Skill Development & Entrepreneurship.
2. It is a web portal with activity-based support material on Mission Chandrayaan for students.
Which of the statements given above is/are correct?
a) 1 only
b) 2 only
c) Both 1 and 2
d) Neither 1 nor 2
Answer: (c)
About ‘Apna Chandrayaan’ Portal
 It is launched by the Ministry of Education and Skill Development & Entrepreneurship.
 The web portal has activity-based support material such as books, quizzes, puzzles, picture builders, and
inspiring stories in the form of graphics on Chandrayaan-3 for students.
 The portal will display inspiring stories in the form of graphic novels depicting events that
 shaped ISRO’s journey to Chandrayaan 3.
 Also, the Digital certificates will be issued to those who would score more than 70%, and the
80
MISSION SHOURYA PSI TEST SERIES -2024
 first 1000 winners will receive age-appropriate books.
 There will be 10 special modules on Chandrayaan-3 which will offer a comprehensive overview of
various scientific, technological, and social aspects.

98. Consider the following statements with respect to Statue of Equality?


1. It is among one of the tallest metallic statues in sitting position in the world.
2. The statue is mounted on a 216-feet high base building named 'Bhadra Vedi'.
3. The Prime Minister will inaugurate the Statue of Equality, on the outskirts of Gujrath.
Which of the above statements is/are correct?
a) 1 only
b) Both 1 and 2
c) Both 1 and 3
d) All of the above.
ANS. a. 1 only.
Explination
Ramanujacharya
 The Prime Minister will inaugurate the Statue of Equality, a statue of Ramanujacharya, on the outskirts
of Hyderabad, Telangana. Hence option c is false.
 India is celebrating his 1,000th birth anniversary as the ‘Festival of Equality’, upholding the view that
the world is one family, ‘vasudhaiva kutumbakam’,”
What Are the Basic Facts About Statue ?
 It is a 216-feet tall statue, which is made of 'panchaloha', a combination of five metals: gold, silver,
copper, brass, and zync.
 It is among one of the tallest metallic statues in sitting position in the world.
 The statue is mounted on a 54-feet high base building named 'Bhadra Vedi'. It has floors devoted for a
vedic digital library and research center, ancient Indian texts, a theater, an educational gallery detailing
many works of Sri Ramanujacharya.Hence option B is false.
Who was Ramanujacharya ?
 Born in 1017 in Sriperumbudur in Tamil Nadu, Ramanujacharya is revered as a Vedic philosopher and
social reformer.
 He was named Lakshmana at the time of his birth. He was also referred to as Ilaya Perumal which
means the radiant one.
 He traveled across India, advocating equality and social justice.
 He revived the Bhakti movement, and his preachings inspired other Bhakti schools of thought. He is
considered to be the inspiration for poets like Annamacharya, Bhakta Ramdas, Thyagaraja, Kabir, and
Meerabai.
 He is famous as the chief proponent of Vishishtadvaita subschool of Vedānta.
 VishishtAdvaita (literally "Advaita with uniqueness; qualifications") is a non-dualistic school of
Vedanta philosophy.
 It is non-dualism of the qualified whole, in which Brahman alone is seen as the Supreme Reality, but is
characterized by multiplicity.
 He went on to write nine scriptures known as the navaratnas, and composed numerous commentaries on
Vedic scriptures.
 Ramanuja’s most important writings include his commentary on the Vedanta Sutras (the Sri Bhasya, or
"True Commentary"), and his commentary on the Bhagavad-Gita (the Gitabhasya, or "Commentary on
the Gita").
 His other writings include the Vedartha Samgraha ("Summary of the Meaning of the Veda"), the
Vedantasara ("Essence of Vedanta"), and Vedantadipa ("Lamp of Vedanta").
 He has also stressed the need of being in tune with nature and not to over-exploit.

81
MISSION SHOURYA PSI TEST SERIES -2024
Why is it called the Statue of Equality ?
 Ramanuja was an advocate of social equality among all sections of people centuries ago, and
encouraged temples to open their doors to everyone irrespective of caste or position in society at a time
when people of many castes were forbidden from entering them.
 He took education to those who were deprived of it. His greatest contribution is the propagation of the
concept of “vasudhaiva kutumbakam”, which translates as “all the universe is one family”.
 He traveled across India for several decades, propagating his ideas of social equality and universal
brotherhood from temple podiums.
 He embraced the socially marginalized and condemned, and asked royal courts to treat them as equals.
 He spoke of universal salvation through devotion to God, compassion, humility, equality, and mutual
respect, which is known as Sri Vaishnavam Sampradaya.
 Ramanujacharya liberated millions from social, cultural, gender, educational, and economic
discrimination with the foundational conviction that every human is equal regardless of nationality,
gender, race, caste, or creed.
Prime Minister has inaugurated the Statue of Unity at Kevadiya in Narmada district of Gujarat.

The Statue of Unity


 The Statue of Unity which is built in honour of Sardar Vallabhbhai Patel is dedicated to the nation.
 Sardar Patel is credited with uniting over 560 princely states in pre-independent India to build the
Republic of India, hence the name of the statue is christened as ‘Statue of Unity’.
 The date for the inauguration of the statue (October 31, 2018) also marks the 143rd birth anniversary of
Sardar Patel.
 It is located on the Sadhu Bet island on the Narmada river, which flows between the Satpura and the
Vindhya mountain ranges.
 The Statue of Unity is the tallest statue in the world. At 182 metres, it is 23 metres taller than China's
Spring Temple Buddha statue and almost double the height of the Statue of Liberty (93 metres tall) in
US.
 The viewing gallery for the statue, at 153 metres, can accommodate 200 people at a time and offer an
expansive view of the Sardar Sarovar Dam.
 The statue is a three-layered structure. The innermost layer is made of reinforced cement concrete
(RCC), comprising two towers 127 metres high that rise till the statue's chest. The second layer is a steel
structure and the third an 8 mm bronze cladding on the surface.
 It was built within three-and-a-half years by more than 3,000 workers, including 300 engineers from
infrastructure major Larsen & Toubro (L&T).
 The Statue of Unity was designed by Padma Bhushan recipient sculptor Ram V Sutar and intricate
bronze cladding work was done by a Chinese foundry, the Jiangxi Toqine Company (JTQ).

99. The chemical name of bleaching powder?


a) Calcium oxide
b) Sodium hydrogen carbonate
c) Sodium bicarbonate
d) Calcium oxychloride
d) Calcium oxychloride
 Bleaching Powder
 Bleaching powder is a pale yellowish powder existing with a strong smell of chlorine.
 It is soluble in water but due to the presence of impurities, we never observe a clear solution.
 Its chemical formula is Ca(OCl2) with its chemical name Calcium hypochlorite.
Common Salt (Sodium Chloride)
 Common salt is formed by the reaction between hydrochloric acid and sodium hydroxide.
 Its chemical name is sodium chloride (NaCl).

82
MISSION SHOURYA PSI TEST SERIES -2024
100. The Archaeological Survey of India (ASI) is an agency functioning under the aegis of which
Ministry?
a) Ministry of Housing and Urban Affairs
b) Ministry of Culture
c) Ministry of Tourism
d) Ministry of Tribal affairs
Ans: (b) Ministry of Culture
Archaeological Survey of India:
 The ASI is the premier organization for the archaeological researches and protection of the cultural
heritage of the country.
 The prime objection of ASI is to maintain the archaeological sites, ancient monuments, and remains of
national importance.
 Headquarters: New Delhi.
 Established: 1861 by Alexander Cunningham.
 It regulates all archaeological activities as per the provisions of the Ancient Monuments and
Archaeological Sites and Remains Act, 1958.
 It functions under the aegis of the Union Ministry of Culture.
 It also regulates Antiquities and Art Treasure Act, 1972.
Cultural Institutions of India
Indira Gandhi National Centre For The Arts - New Delhi
 The India Gandhi National Centre for the Arts (IGNCA) is a leading national institution dedicated to the
advancement of knowledge in the arts and culture, as well as the study of the connections between arts
and culture and other fields of study and elements of life.
 It was established in 1985 in honour of Smt. Indira Gandhi, the late Prime Minister of India.
Nehru Memorial Museum and Library
 The Nehru Memorial Museum and Library (NMML) was designed by Sir Edwin Lutyens for the
imperial capital in 1929-30.
 It was the official house of the last British Commander-in-Chief, and it was known as the Teen Murti
House.
Centre for Cultural Resources and Training
 The Centre for Cultural Resources and Training (CCRT) was established by the Ministry of Culture to
combine education and culture.
 It was founded in 1979 at the request of Dr Kapila Vatsyayan and Smt. Kamala Devi Chattopadhyay.
National Archives of India
 It is one of the earliest British organizations, which was established to retain the administrative records
of the Indian state.
 Prof. G.W. Forrest created the archive, sometimes known as the "Imperial Records Department," in
1891 in the former British capital of Calcutta (now Kolkata).
Indian Council for Cultural Relations
 The Indian Council for Cultural Relations (ICCR) was founded under the Ministry of Culture's auspices
to carry out programs that promote Indian culture on a global scale.
 It was founded in 1950 by Maulana Abul Kalam Azad, who believed strongly in developing cultural
contacts with different countries and civilizations.
Indian Council of Historical Research
 The Indian Council of Historical Research (ICHR) was founded in 1972, following the passage of the
Societies Registration Act.
 It is a self-supporting organization that receives funding from the University Grants Commission
83
MISSION SHOURYA PSI TEST SERIES -2024
(UGC).
 It was established to promote and preserve Indian history. It served as a gathering place for historians to
discuss new ideas and existing projects.
Indian National Trust for Art and Cultural Heritage (INTACH)
 The Indian National Trust for Art and Cultural Heritage, or INTACH, is a non-profit organization based
in New Delhi that was created in 1984.
 The organization's main goal was to raise awareness of our history and encourage people to take action
to preserve it.
 'Chapters' are the numerous branches of the organization.
Sahitya Akademi
 In 1954, the Indian government formed the "National Academy of Letters," or Sahitya Akademi.
 The major purpose of this organization was to serve as a national organization dedicated to promoting
literary culture in India, as well as to cultivate and coordinate literature in all Indian languages and to
promote the country's general national unity.
Sangeet Natak Akademi
 The Government of India established the Sangeet Natak Akademi (SNA) in 1952 as the first national
academy for the arts.
 Its main goal was to establish a setting for Indian music, theatre, and dances.
 It was expected to be the country's major showcase for the performing arts.
 It also had the difficult duty of promoting India's vast intangible legacy, which was manifested in the
forms of music, dance, and theatre.
Lalit Kala Akademi
 The Government of India established the National Academy of Art, also known as the Lalit Kala
Akademi, in 1954 with the primary purpose of promoting fine arts in India.
 The academy is a self-governing organization supported by the Ministry of Culture.
 They promote the appreciation and knowledge of beautiful arts.
 even though they deal with both national and foreign art, they are primarily concerned with the
promotion and preservation of Indian art.

84

You might also like